diff --git "a/Accounting/business_accounting_vol_5_1920.md" "b/Accounting/business_accounting_vol_5_1920.md" new file mode 100644--- /dev/null +++ "b/Accounting/business_accounting_vol_5_1920.md" @@ -0,0 +1,34882 @@ +A black and white image with faint, scattered light spots. + +A circular seal with "UNIVERSITY OF CALIFORNIA" around the top, "LOS ANGELES" at the bottom, and a central design featuring a stylized sun or star. +THE LIBRARY +OF +THE UNIVERSITY +OF CALIFORNIA +LOS ANGELES + +This book is DUE on the last date stamped below. + +A torn edge of a book page. + + +Library +The words "DUE" and "on the last date stamped below" +are printed in red ink. + + +Digitized by the Internet Archive +in 2007 with funding from +Microsoft Corporation + +http://www.archive.org/details/businessaccounti05greeiala + +UNIVERSITY OF CALIFORNIA, +LIBRARY, +LOS ANGELES, CALIF. + +A blank white page. + +White vertical stripe on a white background. + +2 + +A blank page with a white background. + +BUSINESS ACCOUNTING +HAROLD DUDLEY GREELEY, C.P.A., Editor + +Volume I—Theory of Accounts +By Harold Dudley Greeley + +II—Constructive Accounting +By George E. Bennett + +III—Cost Accounting +By DeWitt Carl Eggleston + +IV—Advanced and Analytical Accounting +By Henry C. Cox + +V—Illustrative Accounting Problems +By Charles F. Rittenhouse and Harold Dudley Greeley + +Business Accounting + +Greeley signature + +VOLUME V +ILLUSTRATIVE ACCOUNTING +PROBLEMS + +By +CHARLES F. RITTENHOUSE, B.C.S. +Certified Public Accountant; Professor of Accounting, Boston University; Member of American Institute of Accountants; Member of National Association of Cost Accountants + +and +HAROLD DUDLEY GREELEY, LL.M. +Certified Public Accountant; Member of the New York Bar; Member of National Association of Cost Accountants; Member of American Institute of Accountants; New York State Society of Certified Public Accountants; Formerly, Lecturer on Accounting, Columbia University, New York + +PC logo + +Third Printing + +NEW YORK +THE RONALD PRESS COMPANY +1921 +40010 + +Copyright, 1925, by +THE ROYALD PRESS COMPANY + +All Rights Reserved + +Bus. Admin. +Library + +HF +5621 +G81b +v. 5 + +EDITORIAL PREFACE + +Ten years ago almost any contribution to the literature of accountancy would have been welcomed. Today, however, with the increasing number of excellent publications, it is incumbent upon one who puts forth a new accounting work to justify his action. Much more is it necessary to explain the publication of a set of accounting books. Hence it is desirable to state at the outset the purpose of "Business Accounting" and to outline its scope and general methods of presentation. + +While many books have been published on accounting topics, in almost every case they are unrelated volumes. In some few instances, a volume on accounting has logically followed another by the same author, but with these few exceptions every one published has been written without connection with, or adjustment to, any of those already existing. Under these conditions, the student of accounting, to get any connected and logical knowledge of his subject, must find one of his books here, another there, a third somewhere else, and bridge over the gaps between them as best he may. The process is difficult, and the accounting knowledge he obtains is not always well co-ordinated and logically developed. + +The volumes of "Business Accounting" are intended to meet this situation. They cannot, it is true, provide a course of study in the sense that prescribed readings are recommended, written answers to questions required, and personal instruction given. Neither do they con- +iii + +iv EDITORIAL PREFACE + +stitute an encyclopaedia of unconnected and isolated articles. Rather they are an attempt to present in simple, non-technical language the basic principles of account-keeping and their application to various lines of business, together with general directions for preparing, analyzing, and interpreting accounting statements. + +One who starts at the beginning of Volume I and works faithfully through to the end of Volume IV, and then solves the problems and examines the solutions of Volume V, should acquire some real understanding of the theory and practice of accounts—a knowledge that, supplemented by experience, should enable him successfully to stand the test of practical work in any ordinary business office and furnish a foundation for going as much further into the study of accountancy as he may desire. + +It may be noted in passing that the volumes of "Business Accounting" have been indexed in such a way as to provide many of the features of an encyclopaedia, so that the person desiring the practice on a particular point or accounting ideas of suggestive value in particular lines of industry will be able to use the set to advantage. + +Taking up the volumes of the set in order—Volume I presents the fundamental principles of account-keeping and statement preparation. Upon these basic principles all systems of account are built. Volume II explains the principles governing the development of the simple accounting procedures described in Volume I to meet the needs of more complicated and more extensive systems of financial accounting. Volume III explains in much the same way how the basic principles + +EDITORIAL PREFACE + +have been applied to factory or cost accounting. Having thus traced the fundamental principles into more elaborate financial and cost accounting procedures, Volume IV treats accounting principles and practices which are more advanced than the basic ones described in Volume I. These advanced principles are in most cases subject to differences of opinion, as to their nature or application, among persons qualified to deal with them, and it is for this reason that their discussion is confined to Volume IV. Supplementing the illustrations of accounting principles and statement preparation, there follows in Volume IV a practical discussion of the methods of verifying accounts and statements and of their interpretation and analysis. + +The set closes with Volume V, which gives a number of problems of a practical nature, together with their solutions. The working of these problems will not only clarify the reader's ideas but in many cases will provide models upon which he can base accounting procedures and build statements to meet concrete situations arising in his own work. + +The readers to whom this set will appeal most strongly may be divided roughly into two classes. There will be, on the one hand, business and professional men, bankers, office managers, and other executives who feel the need of understanding in a general way the methods of modern account-keeping and statement preparation. There can hardly be excuse nowadays for them to consider bookkeeping methods and accounting statements as too complicated to understand or of such slight importance as to merit no attention. They need a grasp of the subject so that they may judge for themselves + +v + +vi +EDITORIAL PREFACE + +whether bookkeepers and other persons who keep accounts for them and render statements to them are giving information which is accurate, adequate, and presented in the most intelligible form. The entire tendency of modern business and daily life is toward more exact accounting, of which the accounting requirements of the present income tax legislation are but one indication. Any person having substantial interests at stake should be able to appraise intelligently the stewardship of those to whom his interests are entrusted and the volumes of "Business Accounting" will give him the technical information this demands. + +The other class of persons to whom "Business Accounting" will appeal is composed of those whose duty it is to keep accounts and to prepare statements. They should find in this set an inspiration and an aid to more intensive study, which in turn will result in improved accounting ability and an enhanced wage. The careful and intelligent use of these books will lead beyond question to increased power of service to employer and community. + +HAROLD DUDLEY GREELEY, +Editor, Business Accounting Set. +New York City, +April 1, 1920 + +PREFACE + +Assuming a thorough knowledge of the theory of accounts and the method of their keeping, accounting ability is then developed by the training of the mental faculties in the art of reasoning and of making logical deductions from given premises. The problems in this volume have been collected with this object in view. + +Where the source of a problem is not indicated, it has been either compiled or selected by the authors because they believe that the practice work involved in its solution will prove valuable as a means of impressing important or difficult points of accounting theory on the student's mind; the problems taken from the examination papers of various C.P.A. examination boards have been selected with a view to covering as wide a field of accounting as possible; the problems which relate wholly to cost accounting procedure have been compiled by D. C. Eggleston, M. E., C. P. A., author of Volume III of this series and assistant professor of cost accounting in the College of the City of New York. + +Bookkeeping may be compared to the routine work of the factory operative; accountancy to the designing, planning, and control of this work. Thus, between the routine work of the bookkeeper and the professional work of the accountant the broad distinction may be drawn of a daily task which is endlessly repeated and a task in which the daily problems are never quite the + +vii + +viii +PREFACE + +same. It is possible to impart a working knowledge of bookkeeping by the mechanical process of exercise and drill in the manner of recording business transactions—as the factory operative learns his trade. It is impossible to become an accountant by any system of drill in account-keeping or by any method of grouping like accounting problems and studying the method of their solution. + +It has been the intention of the authors to present a series of problems the solution of which is based on theory of accounting as developed in the preceding volumes; but it is not expected that every difficult point involved in the solution will become clear by referring to the discussion bearing upon that point in the preceding volumes. So far, however, as theory and illustration can be helpful in the working out of problems the solutions of which depend upon the exercise of the analytical and reasoning faculties, this help is afforded by the "tie-up" between each problem and the points of accounting theory which it illustrates—as indicated in the notes which follow the solutions. + +With the "Points Illustrated" to guide him, and with the necessary amount of mental perspiration, the student, it is hoped, will find in this book ample practice work of the kind which develops analytical accounting ability and leads to a mastery of the subject of accountancy. + +CHARLES F. RITTENHOUSE +HAROLD DUDLEY GREELEY + +April 1, 1920 + +Illustrative Accounting Problems + +[API_EMPTY_RESPONSE] + +Illustrative Accounting Problems + +**Problem 1** + +The cash book of a firm at a given date showed a balance available amounting to $10,000. The firm's bank pass-book showed a balance of $10,010. The firm had deposited checks $8,264, which had not been collected by the bank and for which no credit had been given. They had also issued various checks which had not been presented for payment, the numbers and amounts thereof being as follows: + + + + + + + + + + + + + + + + + + + + + + + + + + + + + + + + +
337800.00
331400.00
332750.00
335356.00
336288.00
337841.00
360346.00
361120.00
36225.00
363294.00
+ +Required: +Statement reconciling cash book and pass-book. + +**Solution to Problem 1** + +RECONCILIATION + +Balance, per pass-book............. $10,018.00 +Add---Checks not credited........... $254.99 +$15,274.00 + +3 + +# ILLUSTRATIVE PROBLEMS + +**Deduct—Checks outstanding:** + + + + + + + + + + + + + + + + + + + + + + + + + + + + + + + + + + + + + + + + + + + + + + +
S37$800.00
S51$00.00
S54750.00
S55586.00
S56182.00
S57125.00
S69346.00
S61150.00
S62143.00
S63854.00
3,374.00
+ +Balance, per cash book $10,000.00 + +**Points Illustrated in Problem 1** + +(a) Bank Reconciliation. This solution follows the lines proposed in Volume I, Chapter XX, § 12. + +--- + +**Problem 2** + +The cash book of a general trading concern shows for the month of January, 1912, the following: + +**Receipts** + + + + + + + + + + + + + + + + + + + + +
1912
Jan.4 Collections from customers$ 8,818.68
7""$147.83
10""1,694.87
13""1,292.55
16""1,468.24
43""2,417.14
69""1,698.34
89""1,548.68
31""1,054.27
Total receipts, per cash book:$15,925.48
+ +ILLUSTRATIVE PROBLEMS 5 + + + + + + + + + + + + + + + + + + + + + + + + + + + + + + + + + + + + + + + + + + + + + + + + + + + + + + + + + + + + + + + + + + + + +
Disbursements
Jan.Overdraft on bank810.32
3Sundry checks4,153.27
71,447.83
11926.84
15835.37
19485.32
29647.83
802,477.85
31Balance as shown by cash book5,090.82
$13,923.48
+ +Cash on hand undeposited amounted to $56.23. +A petty cash fund is operated on the imprest system. +The books had been audited to the 31st of December, 1911, and the fact established that the overdraft of $10.32 was correct, after all checks drawn had been presented and paid by the bank. +The deposits in the bank for the month of January, as shown by the bank pass-book, after having it balanced at the close of business January 31, amounted to $13,854.37, and the checks returned by the bank for the same period totaled $8,832.43. +There were checks outstanding at the time of balancing, January 31, amounting to $53.37. +Fraud is suspected on the part of the cashier, and you are asked to check the transactions recorded by him as shown in cash book. + +Prepare a statement showing the results of your investigation. Also state, with reasons, what further documents and records you will require, if any, to trace the cash transactions fully. + +(Ohio C. P. A. Examination.) + +6 +ILLUSTRATIVE PROBLEMS + +**Solution to Problem 2** + +**EXHIBIT A** +BANK STATEMENT AND RECONCILIATION + + + + + + + + + + + + + + + + + + + + + + + + + + + + + + + + + + + + + + + + + + + + + + + +
1912
Jan. 1Debit balance O/D$ 10.32
31Debit total checks cashed8,832.45 $ 8,842.75
Credit deposits15,834.37
Credit balance pass-book8,501.62
Deduct outstanding checks33.47
Company's bank balance8,498.35
Adds.- Cash on hand5.22
Company's actual cash8,501.58
+ +**EXHIBIT B** +RECEIPTS, DEPOSITS, AND DISBURSEMENTS + +Receipts and Deposits: +January cash book receipts $14,092.48 +Less-Cash not deposited 56.23 +Receipts that should have been deposited 813,969.85 +Actual deposits 13,854.37 +* Actual shortage in deposits, January 31 $114.88 + +Cash Book Disbursements: +As footed on cash book in January... $8,883.54 +Less-Discount error................. 10.00 $ 8,873.54 +Bank Disbursements: +Actually paid in January............ $8,892.43 +Checks outstanding as issued........ 53.27 +Disbursements that should appear on cash book $8,883.70 +Disbursements in excess of cash book (Exhibit C) $10.36 + +*Solution by W. P. Kite, C.P.A. + +A table showing a bank statement and reconciliation. + +ILLUSTRATIVE PROBLEMS +7 + +EXHIBIT C + +CASH BOOK—STATEMENT AND CORRECTIONS + + + + + + + + + + + + + + + + + + + + + + + + + + + + + + + + + + + + + + + + + + + + + + + + + + + + + + + + + + + + + + + + + + + + + + + + + + + + + + +
1912
Jan. 31Receipts per footings815,965.48
Add—to correct error in footing100.00
814,065.48
1Balance overdraft810.92
31Disbursements, per footing88,883.34
Deduct—to correct error10.008,873.34
Add—to correct, per checks drawn (Ex-
hibit B)
10.56
31Cash book balance8,896.02
Less—Bank balance and cash on hand (Exhibit A)5,014.58
Net amount of shortage (Exhibit B)3,181.44
+ +Points Illustrated in Problem 2 + +(a) Reconciliation of Bank Balance. The necessity for and method of reconciling bank balances are explained in Volume I, Chapter XX, § 12. + +(b) Verification of Balance on Deposit. The advisability of having the balance on deposit verified by the proprietor or some one entirely in his confidence is pointed out in Volume IV, Chapter XXXII, § 8. + +Problem 3 + +The Universal Company handles its currency disbursements by the imprest system. As of January 1, 1920, the amount of the fund is $1,000. At the close of each month, the petty cashier submits to the treasurer a statement of payments for the month, and receives + +8 +ILLUSTRATIVE PROBLEMS + +the amount required to replenish the fund. The voucher system is not used. + +On January 31, 1920, a statement is received from the petty cashier showing payments of $462.50. These are analyzed by the bookkeeper as being chargeable to the following accounts: + + + + + + + + + + + + + + +
General Administrative Expense$212.60
Selling Expense191.40
Manufacturing Expense58.50
+ +It is decided as of this date to reduce the fund by $200. +Show entry or entries for accomplishing the above. + +Solution to Problem 3 + +On the credit side of the cash book a check for $462.50 should be entered and charged as follows: + + + + + + + + + + + + + + +
General Administrative Expense$212.60
Selling Expense191.40
Manufacturing Expense58.50
+ +This check should be cashed and the money placed in the petty cash fund, thus restoring the fund to $1,000. Thereupon $200 should be withdrawn from the fund and deposited in the bank. An entry to cover this item should be made on the debit side of the cash book, the credit being to the Petty Cash account. + +Points Illustrated in Problem 3 + +(a) Imprest Petty Cash Fund. This problem illustrates the operation of an imprest petty cash fund as described in Volume I, Chapter XX, § 15. + +ILLUSTRATIVE PROBLEMS 9 + +**Problem 4** + +A manufacturing concern buys its raw product from comparatively few dealers. A simple purchase journal has been in use since the company began business, in which only invoices of stock purchased are recorded. Accounts with individual creditors are kept in the general ledger. + +In revising their accounting methods it is decided to install a voucher system and to discontinue the accounts with individual creditors. This change takes place as of July 1, 1919. On that date, the following creditors' accounts appear in the trial balance taken from the general ledger: + + + + + + + + + + + + + + + + + + + + + + +
Thomas Groom and Son$ 496.85
R. W. Irwin and Company1,138.00
George Newton Company914.55
Begg and Beggz, Inc.697.50
General Chemical Company48.90
+ +Make such entries and accompanying comments as would be necessary to give effect to the change. + +**Solution to Problem 4** + +In order to install the voucher system, a voucher register must be opened and the first entry in it will be to record the five following vouchers: + + + + + + + + + + + + + + + + + + + + + + +
Thomas Groom and Son$ 496.85
R. W. Irwin and Company1,138.00
George Newton Company914.55
Begg and Beggz, Inc.697.50
General Chemical Company48.90
+ +These vouchers should be numbered and entered in the vouchers payable column, but they should not be + +10 +ILLUSTRATIVE PROBLEMS + +distributed to purchase or expense accounts. They should be entered in the sundries column of the voucher register, to be debited to the individual accounts pay-able with these various creditors now appearing on the ledger. + +**Points Illustrated in Problem 4** + +(a) Controlling Account. The purpose and use of controlling accounts are explained in Volume I, Chapter XXIV, § 4. +(b) Voucher System. The theory underlying the voucher sys-tem and a general description of Its methods of operation, together with forms, are explained in Volume I, Chapter XXII. + +--- + +**Problem 5** + +A mercantile concern which began in a small way and with a few customers has always kept its accounts with customers in the general ledger. The number of such accounts has increased to such an extent that it is now desired to transfer all accounts with customers to a loose-leaf sales ledger and to establish a controlling account therewithin in the general ledger. + +Using the following customers' balances as of January 1, 1920, show how the transfer of accounts and the opening of the control account would be effected: + + + + + + + + + + + + + + + + + + + + + + +
James Conway and Company$ 63.99
Henry Jones197.63
Thomas W. Small418.15
Charles W. Cutter94.59
L. W. Hunter and Company147.90
+ +ILLUSTRATIVE PROBLEMS 11 + +**Solution to Problem 5** + +To close out the customers' accounts in the general ledger, and to set up a controlling account for a sales ledger, the following entry would be made in the journal: + + + + + + + + + + + + + + + + + + + + + + + + + + +
Accounts Receivable8716.14
James Conway and Company$ 83.90
Henry Jones197.63
Thomas W. Smal812.13
Charles W. Cutter94.50
L. W. Hunter and Company147.96
+ +**Points Illustrated in Problem 5** + +(a) Need for Controlling Account. This problem illustrates the need for controlling accounts in order to insure convenience and speed in handling the numerous details. For a general discussion of the functions of controlling accounts, see Volume I, Chapter XXIV, § 3. + +(b) Opening Entry. The entry opening the controlling account is in accordance with the plan outlined in Volume I, Chapter XXIV, § 4. + +**Problem 6** + +Brown has a customers ledger, a purchase ledger, and a general ledger, the latter containing controlling accounts with the other two. When his bookkeeper submitted to him trial balances of the three he observed that White owed him $100, subject to a cash discount of $2\frac{1}{2}\%$, and an allowance for outward freight of $81.68$, neither of which items had been entered in the books; and that he owed White $100$, subject to a discount of + +12 + +**ILLUSTRATIVE PROBLEMS** + +4%, which had not been entered. He directed the bookkeeper to adjust the accounts by a remittance of stamps. Draft entry or entries that will close the two personal accounts and maintain the reconciliation of the ledgers. Separate accounts are kept for customers' discount and purchase discount. + +**(Massachusetts C. P. A. Examination.)** + +**Solution to Problem 6** + + + + + + + + + + + + + + + + + + + + + + + + + + +
Accounts Payable (White)$100.00
Discounts on Sales2.50
Freight and Carriage Out1.68
Accounts Receivable (White)$100.00
Discounts on Purchases4.00
Postage.18
+ +(Explanation to set forth all the details of the entry.) + +**Points Illustrated in Problem 6** + +(a) Unusual Transactions. As pointed out in Volume I, Chapter XXIV, § 13, care must be exercised in recording unusual transactions such as the one in this solution, to make sure that the controlling account is adjusted for every item posted in any detail account. If this procedure is not followed, the controlling account will be thrown out of balance with the subsidiary ledger. + +(b) Cash Discounts. For a discussion of cash discounts and the proper method of recording them, see Volume I, Chapter XXV. + +(c) Anticipation of Cash Discounts. Cash discounts depend entirely on promptness of payment and thus are earned and should be recorded only when taken at the time of payment. In this solution the cash discounts are taken by the settlement of the mutual accounts. (See Volume I, Chapter XXV, § 8.) + +A table showing Accounts Payable (White), Discounts on Sales, Freight and Carriage Out, Accounts Receivable (White), Discounts on Purchases, Postage. + +ILLUSTRATIVE PROBLEMS 13 + +**Problem 7** + +A and Company sold to B and Company a bill of goods for $1,000—terms 2% 10 days, net 30 days. + +B and Company did not take advantage of the cash discount, but, when the invoice was due, offered in settlement cash $833.34 and two notes for $833.33 each, one at 60 days and one at 90 days, due at their bank. + +A and Company accepted this settlement, and carried the notes for 30 days. They then discounted them at their bank at 5%. + +B and Company paid the 60 days’ note when due. + +Three days before the 90 days’ note was due, A and Company received from B and Company a 30 days’ note for $200 due at B and Company’s bank, and a check for the balance plus the discount on the new note at 6%, with a request that A and Company take care of the 90 days’ note, which they agreed to do. + +A and Company discount the new note at 5% at their bank when they take up the 90 days’ note. + +The $200 note is paid by B and Company when due. + +Journalize the above transactions for A and Company. + +(Pennsylvania C. P. A. Examination.) + +**Solution to Problem 7** + +The transactions as specified in the above question can be expressed by means of the following journal entries: + +*Solution by E. F. Money, C. P. A.* + +--- +14 + +14 +ILLUSTRATIVE PROBLEMS + + + + + + + + + + + + + + + + + + + + + + + + + + + + + + + + + + + + + + + + + + + + + + + + + + + + + + + + + + + + + + + + + + + + + + + + + + + + + + + + + + + + + + + + + + + + + + +
B and Company$1,000.00
Sales$1,000.00
Sold bill of goods 8/10, net 30.
Cash353.94
Notes Receivable666.66
B and Company1,000.00
Bank Discounted666.49
Cash Discounted4.17
Bank Discounted4.17
Notes Receivable Discounted666.66
Discounted notes of B and Company at bank. Rate of discount, 5%.
Notes Receivable Discounted353.93
Note Receivable353.93
Note of B and Company due today and paid by maker.
Cash154.35
Notes Receivable-$90.00
Notes Receivable Discounted353.93
Cash Received from B and Company cash and new note in renewal of their 90-day note.1.00
Note Receivable Discounted353.93
Cash Discounted-83
Note Receivable Discounted-83
Cash Discounted-83
Cash Received from B and Company's note due today and discount received from B and Company.154.16
Points Illustrated in Problem 7
(a) Cash Discounts. For a discussion of the nature of cash discounts and the proper method of recording them, see Volume I, Chapter XXV.
Cash Discounts.Description of the nature of cash discounts and the proper method of recording them, see Volume I, Chapter XXV.
+ +ILLUSTRATIVE PROBLEMS 15 + +(b) Time of Taking Discount. See Volume I, Chapter XXV, § 8, for a discussion concerning the time when cash discounts become effective and may properly be recorded. + +(c) Provision for Future Cash Discounts. As pointed out in Volume I, Chapter XXV, § 10, at the time a balance sheet is prepared it is sometimes advisable to provide for possible losses due to the taking of cash discounts by customers. + +--- + +**Problem 8** + +A corporation has been accustomed to charge the purchase of machinery to the Machinery account at cost, and each year to charge the Manufacturing account and to credit a Reserve for Depreciation account with an amount which will offset the cost of the machinery by the time it is estimated that it will be advisable to scrap the machines. During the period that you have been employed to audit the account, you find that the corporation has sold two machines for $300 each, and this amount has been credited to the Machinery account. One of them cost $1,000, and the amount reserved for depreciation on this machine is $600. The other cost $1,500, and the amount reserved for depreciation is $850. + +Make the adjusting entries to correct the books. + +Solution to Problem 8 + + + + + + + + + + + + + + + + + + +
Reserve for Depreciation$600.00
Machinery$ 500.00
Surplus100.00
Machine costing $1,000 sold for $300, on which $600
+ +16 + +**ILLUSTRATIVE PROBLEMS** + +had been reserved. Entry previously made: Cash, debit, to Machinery, credit, for $800. + +| Reserve for Depreciation | 850.00 | +| Surplus | 150.00 | +| Machinery | 1,000.00 | + +Machine costing $1,000 sold for $800, on which $820 had been recorded as charged against Surplus. Entry previously made: Cash, debit to Machinery, credit, for $850. + +**Points Illustrated in Problem 8** + +(a) Bookkeeping for Depreciation Reserves. The method of recording depreciation reserves is discussed in Volume I, Chapter XXIX, § 4. + +(b) Bergman's Rule. The bookkeeping to record replacements is discussed in Volume I, Chapter XXXIX, § 5. + +(c) Surplus Adjustment. The necessity for adjusting surplus, capital, undivided profits, or other similar accounts to record items applicable to prior fiscal periods is discussed in Volume IV, Chapter VII, § 4. + +--- + +**Problem 9** + +A machine costing $10,000 was estimated to have a life of ten years, with a residual value of $1,000. At the close of each year a charge of $900 was made and a similar amount credited to Reserve for Depreciation. Just prior to closing the books at the end of the tenth year the machine was discarded and sold, bringing $2,000, and a similar machine was bought costing $15,000. Give the journal entries that you would make to close the books at the end of the tenth year in order + +ILLUSTRATIVE PROBLEMS 17 + +to cover these transactions and to make necessary adjustments. Interest is not to be calculated. + +**Solution to Problem 9** + + + + + + + + + + + + + + + + + + + + + + + + + + + + + + + + + + + + + + + + + + + + + + + + + + + + + + + + + + + + + + + + + + + + + + + + + + + + + + + + + + + +
Depreciation8900.00
Reserve for Depreciation8900.00
To set up the 10th and last annual reserve for depreciation.
Cash8,000.00
Machinery2,000.00
To show sale of old machine for cash.
Reserve for Depreciation8,000.00
Machinery8,000.00
To charge the loss due to replacement to the reserve account:
Reserve for Depreciation1,000.00
Surplus1,000.00
To restore to Surplus the amount of reserve in excess of the actual loss due to replacement:
Machinery13,000.00
Cash15,000.00
To show purchase of new machinery.
+ +**Points Illustrated in Problem 9** + +(a) Depreciation Reserves. The nature of depreciation reserves and the bookkeeping necessary to record them are outlined in Volume I, Chapter XXIX, § 4. + +(b) Retirement of Assets. The bookkeeping procedure to record the retirement of assets when their useful life has ceased is explained in Volume I, Chapter XVIII, § 5. + +(c) Surplus Adjustments. In Volume IV, Chapter VII, § 4, the necessity for adjusting surplus or other similar account to record items applicable to prior fiscal periods is discussed. + +18 + +**ILLUSTRATIVE PROBLEMS** + +(d) Straight-Line Depreciation. The depreciation in this prob- +lem was calculated on the straight-line method. A de- +scription of this method and arguments for and against +its use are outlined in Volume I, Chapter XXXIX, § 7, and +in Volume IV, Chapter XII, § 7. + +--- + +**Problem 10** + +An account with Reserve for Depreciation of De- +livery Equipment showed on December 31, 1919, a balance of $949.80. The Delivery Equipment account of the same date showed a balance of $13,968.40. + +In August, 1919, a horse died which cost $300, no entry being made at the time. Three years' depreciation had already been provided for at the time of the horse's death at the rate of 10% per annum, based on cost. + +In October, 1919, a horse which cost $275 was sold for $175, the difference between cost and selling price having been charged to the reserve account. This horse was bought at the same time the other one was and the same depreciation has been provided for. + +Make necessary adjustments. + +--- + +**Solution to Problem 10** + + + + + + + + + + + + + + +
Reserve for Depreciation of Delivery Equipment$ 90.00
Surplus$10.09
Delivery Equipment$300.00
+ +To adjust loss caused by death of a horse: cost $200; +depreciation 10% for three years $60. + +ILLUSTRATIVE PROBLEMS 19 + +Surplus 17.50 +Reserve for Depreciation of Delivery Equipment 17.50 + +To correct a previous entry charging $100 to reserve when only $82.30 had been reserved. Cost of horse $67.75, and for $195, three years' depreciation at 19% $84.30. + +Points Illustrated in Problem 10 + +(a) Replacements and Retirements. The accounting procedure for recording replacements and retirements is discussed in Volume I, Chapter XXIX, § 8 and 5. +(b) Surplus Adjustment. The necessity for recording any surplus adjustment during the fiscal year and the fiscal periods is discussed in Volume IV, Chapter VII, § 4. +(c) Straight-Line Depreciation. Depreciation in this problem has been calculated on the straight-line method. This method is discussed in Volume I, Chapter XXIX, § 7, and Volume IV, Chapter XII, § 7. + +Problem 11 + +December 31, 1918, at the time of closing the books, the Henry Hudson Company set aside 1½% of accounts receivable as a reserve for bad debts. The accounts receivable on that date showed a balance of $62,747.93. +June 30, 1919, the accounts of R. W. Rollins and Company for $137.20 and of J. C. Cutter for $42.25 are written off, as repeated attempts have been made to collect them. +September 1, 1919, a final dividend of 20% from trustees in bankruptcy for Thomas Knight on a claim of + +20 + +ILLUSTRATIVE PROBLEMS + +$638.20 was received. Previous to this, dividends of 30% and 20% had been received. + +July 2, 1920, J. C. Cutter paid us in full the amount written off on June 30, 1919. + +Make necessary entries. + +**Solution to Problem 11** + + + + + + + + + + + + + + + + + + + + + + + + + + + + + + + + + + + + + + + + + + + + + + + + + + + + + + + + + + + + + + + + + + + + + + + + + + +
Profit and Loss8941.82
Reserve for Bad Debts8941.82
15% of accounts receivable (867,747.93) set aside as a reserve for bad debts.
Reserve for Bad Debts179.45
Accounts Receivable179.45
Accounts Receivable worthless:
R. W. Rollins & Co.8137.80
J. C. Cutter44.85
Cash187.64
Reserve for Bad Debts191.46
Accounts Receivable319.10
Final dividend due from trustees of Thomas Knight -claim $635.80, previous dividends, 30% and 20%
Balance 30% charged off.
J. C. Cutter44.85
Surplus44.85
Cutter pays amount previously written off; above en- try is to reopen his account.
Cash44.85
J. C. Cutter received a J. C. Cutter amount previously written off as uncollectible.44.85
+ +**Points Illustrated in Problem 11** + +(a) Provision for Bad Debts. In this problem the provision for bad debts was based on the amount of accounts receivable outstanding. It is more customary to base this provision + +ILLUSTRATIVE PROBLEMS 21 + +on the volume of sales. This matter is discussed in Volume I, Chapter XXIX, § 11. + +(b) Charging Off Accounts. The procedure involved in charging uncollectible accounts against the reserve for bad debts is covered in Volume I, Chapter XXIX, § 11. + +(c) Collection of Accounts Previously Written Off. The book-keeping required upon the collection of an account pre-viously written off as uncollectible is described in Volume IV, Chapter VII, § 4. + +--- + +**Problem 12** + +French and Dysart, Inc., are engaged in the manu- +facture of lathes. At the time of closing the books June 30, 1920, the following facts are discovered by the accountant: + +March 1, the Essex Machine Company ordered the delivery of two lathes, which had been manufactured for them and had been held for shipping instructions since September, 1919. The lathes had been charged to the Essex Machine Company on September 21, 1919, but by an oversight were included in the inventory taken December 31. One machine was billed at $962.50 and the other at $750. + +The inventory taken December 31 was also found to contain the following clerical errors: finished stock, $3,100 too raw; raw materials, $1,000 too little. + +L. P. Fuller, a customer of the company, failed and his affairs were settled in the bankruptcy court. A final dividend was received March 1, 1919, leaving an unpaid balance of $610 which was charged off to Profit and Loss. Fuller began business anew and desiring to + +23 + +**ILLUSTRATIVE PROBLEMS** + +make settlement in full with all creditors as he is able to do so, sends the company his check for $800 on account on June 30, 1920. What entry should the bookkeeper make? + +If the $610 had been charged to a Reserve for Doubtful Accounts, what entry would you advise at the time of recovering the $800? + +**Solution to Problem 12** + +**ADJUSTMENTS** +June 30, 1920 + + + + + + + + + + + + + + + + + + + + + + + + + + + + + + + + + + + + + + +
Surplus$1,712.50
Inventory$1,712.50
To adjust error in including two lathes billed to the Essex Machine Company in the inventory of December 31, 1919.
Surplus$2,100.00
Inventory$2,100.00
To adjust net error in figuring inventory of December 31, 1919:
Finished stock85,100 too much
Raw materials1,900 too little
$2,100 net error
+ + + + + + + + + + + + + + +
Accounts Receivable610.00
Surplus610.00
To bring on balance of claim against L. P. Fuller (charged on March 5, 1919) who desires to make a settlement.
+ + + + + + + + + + + + + + +
Cash300.00
Accounts Receivable300.00
L. P. Fuller on account.
+ +If the $610 had been charged off against a reserve the entries would be as follows: + +ILLUSTRATIVE PROBLEMS 23 + + + + + + + + + + + + + + + + + + +
Accounts Receivable$ 610.00
Reserve for Bad Debts$ 610.00
Cash300.00
Accounts Receivable300.00
+ +**Points Illustrated in Problem 12** + +(a) Surplus Adjustment. In Volume IV, Chapter VII, § 4, will be found a discussion of the adjustments of surplus or other similar account required upon the entry of items affecting prior fiscal periods. + +(b) Collection of Account Charged Off. The adjustment necessary when an account previously charged off has been collected is discussed in Volume IV, Chapter VII, § 4. + +--- + +**Problem 13** + +The following facts are recorded in a Merchandise account in A's ledger: + + + + + + + + + + + + + + + + + + + + + + + + + + + + + + + + + + +
Sales$45,000.00
Purchases30,000.00
Freight and Cartage (on sales $300, on purchases $300)1,000.00
Returns by Us.3,500.00
Cash Discounts Allowed Customers575.00
Cash Discounts Taken by A750.00
Allotances to Customers1,000.00
Inventory at the close of the year is valued at5,085.00
+ +(a) Set up the Merchandise account as it now stands in A's ledger, after which make journal entry or entries which will close the old Merchandise account and open such new accounts as you think necessary to show a complete classification of the trading activities of A. + +24 + +ILLUSTRATIVE PROBLEMS + +(b) Submit a statement showing the profit on merchandise sales. +(c) Make entries for closing all the accounts into a Trading account. + +(Wisconsin C.P.A. Examination.) + +**Solution to Problem 13** + + + + + + + + + + + + + + + + + + + + + + + + + + + + + + + + + + + + + + + + + + + + + + + + + + + + + + + + + + + + + + + + + + + + + + + + + + + + + + + + + + + + +
MERCHANDISE
Purchases$30,000.00Sales$45,000.00
Freight and Cartage In.500.00Purchase Returns2,500.00
Freight and Cartage Out.500.00Cash Discounts on Pur.
Cash Discounts on Sales6,750.00Cash Discounts on Pur.750.00
Sales Allowances1,600.00
Purchases$30,000.00
Freight and Cartage In.500.00
Freight and Cartage Out.500.00
Cash Discounts on Sales6,750.00
Sales Returns and Allowances1,800.00
Sales$45,000.00
Purchase Returns and Allowances2,500.00
Cash Discounts on Pur.750.00
+ +The above entry is to close out the Merchandise account and to set up instead separate accounts showing the various trading activities. + +**STATEMENT OF PROFIT** + + +
Sales:$45,000.00
Gross Sales:
Deduct - Returns and Allowances:1,800.00
Net Sales:$44,258.88$44,258.88
Cost of Goods Sold:
Purchases (including opening inventory, if any):$39,528.88$39,528.88
Add- Freight and Cartage In:258.88258.88
Total Cost of Purchases:$39,787.76$39,787.76
+ +A table showing Merchandise transactions and Statement of Profit calculations. + +ILLUSTRATIVE PROBLEMS 25 + + + + + + + + + + + + + + + + + + + + + + +
Deduct—Returns and Allowances$2,500.00
Net Purchases$28,000.00
Deduct—Inventory at close of year5,000.00
Cost of Goods Sold23,000.00
Gross Profit on Sales821,000.00
+ +JOURNAL ENTRIES + + + + + + + + + + + + + + + + + + + + + + + + + + + + + + + + + + + + + + + + + + + + + + + + + + + + + + + + + + + + + + + +

































































































<
Sales8,100.00
Sales Returns and Allowances8,100.00
To show net sales in Sales account.
Purchases500.00
Freight and Cartage In500.00
To show total cost of purchases in Purchases ac-
counts.
Purchase Returns and Allowances$2,500.00
Purchases$2,500.00
To show net purchases in Purchases account.
Sales44,000.00
Trading Account44,000.00
Trading Account28,000.00
Purchases28,000.00
To transfer balances of Sales and Purchases ac-
counts.
+ +Merchandise Inventory +Trading Account +To record inventory at close of year. + +Points Illustrated in Problem 13 + +(a) Interpretation of Problem. The problem requires the clos- +ing of "all the accounts into a Trading account." In view +of the fact that some of the accounts listed do not properly +belong in a Trading account, it is assumed that the prob- + +26 + +**ILLUSTRATIVE PROBLEMS** + +(a) Inadequate Merchandise Account. This problem illustrates the old-fashioned type of Merchandise account which con- +tains all the elements from which the gross profit is determined. Such an account is inadequate, for reasons explained in Volume I, Chapter VIII, § 3. + +(b) Separate Gross Profit Account. The desirability of having a separate account for each element from which the gross profit is determined is explained in Volume I, Chapter VI, § 13. + +(c) Determination of Gross Profit. The method of calculating gross profit as exemplified in this solution is described in Volume I, Chapter VIII, § 4. + +(d) Cash Discounts. Cash discounts form no part of the trading income or expense, in accordance with principles explained in Volume I, Chapter XXV, § 8. + +(e) Freight and Cartage Inward. This is an addition to the cost of purchases, as described in Volume I, Chapter IX, § 8. + +(f) Freight and Cartage Outward. This item does not appear in the Trading account because it is not an expense to be considered in determining the gross profit. For a discussion of it see Volume I, Chapter IX, § 5. + +(g) Bookkeeping to Facilitate Statement Preparation. The accounts are kept in such a way as to facilitate the preparation of financial statements. The desirability of this is pointed out in Volume I, Chapter VI, § 9. + +(h) Account for Merchandise Inventory. A separate account is opened for the merchandise inventory, as suggested in Volume I, Chapter VIII, § 4. This account may be kept on the ledger during the next fiscal period, being closed into Merchandise Purchases or into Trading account at the end of the period; or it may be closed into Purchases at the beginning of the next fiscal period. Instead of opening a + +A table showing various accounting items and their descriptions. + +ILLUSTRATIVE PROBLEMS 27 + +separate account for merchandise inventory, the amount of the inventory may be brought down as a balance of the Purchases account. The journal entry to effect this would be Purchase (net) (or Cost of Goods Sold), respectively. + +(j) Closing Entries. The preparation of closing entries in the journal is described in Volume I, Chapter VIII, § 7. + +(k) Trading Account. The purpose of a Trading account is to show the gross profit on the books. This account is a subdivision of the Profit and Loss account, to which its balance is transferred. + +--- + +**Problem 14** + +A, B, and C agree to start in business with a capital of $200,000, of which A is to furnish $100,000 and B and C $50,000 each. A is to have $\frac{1}{2}$ interest in the business and B and C each $\frac{1}{4}$. Interest at 5% is to be credited on excess, or charged on deficiency of capital. A contributes $100,000, B $45,000, and C $40,000. How would the capital accounts stand on the books after adjusting the interest at the end of the year? + +(New York C. P. A. Examination.) + +**Solution to Problem 14** + +**First Solution** + +B is charged interest on his deficiency of $5,000, resulting in a debit of $250 to his account and a credit to Profit and Loss. + +C is charged interest on his deficiency of $10,000, resulting in a debit of $500 to his account and a credit to Profit and Loss. + +28 + +**ILLUSTRATIVE PROBLEMS** + +The total credit to Profit and Loss is divided, A ½; B and C ¼ each, leaving A with a credit of $100,375; B $44,937.50; C $39,687.50. + +**Second Solution** + +The total capital contributed is $185,000; had this been contributed in such proportion as to give A a ½ interest, B ¼, and C ¼, the investments of each would have been: A $92,500; B $46,250; and C $46,250. Instead, A contributed $87,500 more than his share; B $1,250 less, and C $6,250 less. Credit A with interest on this excess and charging B and C with interest on their deficiencies, we arrive at the results shown in the solution above. + +**Points Illustrated in Problem 14** + +(a) Division of Profits. As pointed out in Volume I, Chapter XXXI, § 6, any basis for the division of profits may be adopted by the partners. In this problem a percentage basis was adopted which was to prevail regardless of the capital contributions. + +(b) Interest on Capital. The expedient for compensating a partner for a capital contribution in excess of that by another partner by means of allowing interest on capital is discussed in Volume I, Chapter XXXI, § 9. + +--- + +**Problem 15** + +D. E. Draper and H. M. Monroe are partners in a wholesale dry goods business. Among other things, the partnership agreement states that 5% interest is to + +ILLUSTRATIVE PROBLEMS 29 + +be allowed each partner on his investment and 6% charged on all drawings in excess of salary allowances, after which the net profit is to be divided as follows: +Draper, two-thirds; Monroe, one-third. + +On January 1, 1920, Draper's capital account showed a credit of $18,512.00, and Monroe's a credit of $12,678.52. During the year their drawings in excess of salary were as follows: + + + + + + + + + + + + + + + + + + + + + + + + + +
DraperMonroe
February 23$ 600.00January 31$1,000.00
May 261,000.00March 1500.00
October 15900.00September 12,000.00
+ +(a) Make entries to give effect to the interest clause of the agreement and to show the distribution of the remaining profit. +(b) Which partner would lose in this case, in the absence of an interest clause? + +Solution to Problem 15 + +(a) + + + + + + + + + + + + + + + + + + + + + + + + + + + + + + + + + + + + + + + + + + + + + + + + + + + +
Interest on Capital$1,559.36
D. E. Draper, Capital Account8935.63
H. M. Monroe, Capital Account*635.93
Interest on capital for one year at 5%
D. E. Draper, Capital Account78.03
Interest on Capital78.03
Interest on Drawings at 5% as follows:
8 600 for 10 months at 8 days$30.80
1,000 " " " " " " " " " " " " " " " " " " " " " " " " " " " "55.83
900 " " " " " " " " " " " " " " "11.40
$78.03 + +A table showing interest calculations for D.E. Draper and H.M. Monroe. + +30 +ILLUSTRATIVE PROBLEMS + + + + + + + + + + + + + + + + + + + + + + + + + + + + + +
H. M. Monroe, Current Account181.00
Interest on Capital181.00
Interest on drawings at 6% as follows:
$1,800 for 5 days$ 56.00
500 " 9 " 30 "82.00
$,000 " 3 " 30 "40.00
$121.00
+ +The balance of the Interest on Capital account as brought on the books by the above entries is $1,360.53, of which Draper is charged with two-thirds and Monroe with one-third. The net result is a debit in Draper's account and a credit in Monroe's, amounting to $89.42. + +The analysis of each account, so far as the interest entries are concerned, follows: + +DRAPER, CAPITAL ACCOUNT + + + + + + + + + + + + + + + + + + + + + + + + + + + + + + +
Debits:
Interest on Drawings878.63
Two-thirds of debit balance of Interest Account907.02
Total895.05
Credit:
Interest on Capital925.63
Net debit839.42
+ +MONROE, CAPITAL ACCOUNT + + + + + + + + + + + + + + + + + + + + + + + + + + + + + + +
Credit:
Interest on Capital:$653.93
Debits:
Interest on Drawings:$181.00
One-third of debit balance of Interest Account:455.51
Total274.51
Net credit859.42
+ +ILLUSTRATIVE PROBLEMS 31 + +(b) + +Monroe would have suffered if the agreement had contained no interest clause, because his investment is more than one-third of the total capital, whereas he is entitled to only one-third of the profits. + +Points Illustrated in Problem 15 + +(a) Interest on Capital. The nature of interest on capital and the correct methods of recording it are outlined in Volume I, Chapter XXXI, §§ 9-11. +(b) Interest on Drawings. The connection between interest on capital and interest on drawings is pointed out in Volume I, Chapter XXXI, § 10. As this matter is commonly misunderstood, it is suggested that this reference be examined with considerable care. + +--- + +Problem 16 + +A and B form a partnership, A investing $30,000 and B $50,000. They agree to share profits and losses equally. They further agree to and do leave their original investments intact. At the end of the first year, the profits from the operation of the business amount to $30,000, against which A has drawn in twelve equal monthly instalments on the last day of each month an aggregate amount of $9,000; B has drawn against his profits on the last day of each quarter the total sum of $2,500. + +Prepare journal entries adjusting interest at 8% per annum in respect to both investments and drawings; + +32 +ILLUSTRATIVE PROBLEMS + +and render statements showing what each partner has in the business at the end of the year. + +**Solution to Problem 16** + + + + + + + + + + + + + + +
Interest on Capital$4,000.00
A. Capital$1,500.00
B. Capital2,500.00
+ +To credit each partner with interest on his capital at 5%: + + + + + + + + + + +
A. Capital$06.85
Interest on Capital206.85
+ +To charge A with interest on drawings at 5% as follows: $750 for the first month, $750 for the last day of each month amount to a monthly drawing of $750; the first drawing would be charged interest for 11 months, the second, 10 months, etc., making an interest charge on $750 for a total of 66 months or 5½ years, or $496.45. + + + + + + + + + + +
B. Capital187.50
Interest on Capital187.50
+ +As B drew $2,500 on the last day of each quarter, he would be charged interest on the first drawing for 9 months, on the second for 8 months, on the third for 7 months, making a total of 18 months' interest charge on $2,500 at 5% or $197.20. + +The account with Interest on Capital would be closed into Profit and Loss, reducing the balance of such account to $26,393.75, which would be closed out by the following entry: + + + + + + + + + + + + + + +
Profit and Loss$26,393.75
A. Capital$13,196.87
B. Capital13,196.88
+ +The following statements show each partner's interest in the business at the end of the year. + +ILLUSTRATIVE PROBLEMS +33 + +A + + + + + + + + + + + + + + +
Investment$80,000.00
Less—Drawings5,000.00
Net Investment$75,000.00
+ +Add: +Interest on Capital 8 $1,500.00 +One-half Net Profit 13,196.88 $14,696.87 +Less—Interest on Drawings 106.85 14,490.62 +Net Capital, December 31. $85,490.62 + +B + + + + + + + + + + + + + + +
Investment$50,000.00
Less—Drawings10,000.00
Net Investment$40,000.00
+ +Add: +Interest on Capital 8 $2,500.00 +One-half Net Profit 13,196.88 $13,696.88 +Less—Interest on Drawings 187.50 15,569.38 +Net Capital, December 31. $85,569.38 + +Points Illustrated in Problem 16 + +(a) Basis of Profit Distribution. In this problem profits and losses are shared equally by agreement between the partners. The various bases upon which profits are usually divided and the law which governs in case no provision is made are discussed in Volume I, Chapter XXXI, §§ 6 and 7. + +(b) Interest on Capital. The net effect of the interest on capital in this problem was to increase B's capital by $509.38, with a corresponding reduction in A's capital. This provides extra compensation to B for his excess capital contribution. This matter is discussed in Volume I, Chapter XXXI, § 11. + +I + +34 +ILLUSTRATIVE PROBLEMS + +**Problem 17** + +Wilson and Lawson are partners, sharing profits and losses equally. The partnership is dissolved December 31, 1919, at which time Wilson's capital investment was $10,000, and Lawson's $8,500. The total liabilities of the firm are $25,000 which includes $5,000 due Wilson on loan account and $2,500 due Lawson on loan account. The assets of the firm are disposed of for $830,-000 on May 1, 1920. Prepare accounts closing the partnership and showing the position in which the partners stand to each other. No allowance for interest is required. + +**Solution to Problem 17** + +BALANCE SHEET OF WILSON AND LAWSON +December 31, 1919 + + + + + + + + + + + + + + + + + + + + + + + + + + + + + + + + + + + + + + +
Cash$30,000.00Wilson Capital$10,000.00
Profit and Loss7,500.00Lawson Capital8,500.00
Trade Creditors17,500.00
Wilson's Loan Account5,000.00
Lawson's Loan Account8,500.00
$37,500.00$37,500.00
+ +The net loss of $7,500 is divided equally between Wilson and Lawson, leaving Wilson's account with a credit of $6,250, and Lawson's account with a debit of $1,250. + +The amount due outside creditors is paid, after which Wilson's loan of $5,000 is paid, leaving a cash balance of $7,500. Inasmuch as Lawson's capital + +ILLUSTRATIVE PROBLEMS 35 + +account has been overdrawn $1,250, Wilson should insist that $1,250 of the amount due Lawson on his loan should be withheld to cancel his deficit. Wilson would take $6,250 of the remaining cash balance, and Lawson, $1,250, closing all the accounts. + +**Points Illustrated in Problem 17** + +(a) Distribution of Loss. The loss in this case was distributed between partners on their profit and loss sharing basis. This is in accordance with principles explained in Volume I, Chapter XXXII, § 6. + +(b) Distribution of Assets. The asset cash was distributed between partners in the amounts called for by their capital accounts. The reason for this is stated in Volume I, Chapter XXXII, § 6. + +(c) Offsetting Losses and Deficit. As explained in Volume I, Chapter XXXII, § 9, the deficit in one partner's account should be offset by his credit for loans. + +--- + +**Problem 18** + +Brown, Smith, and Jones are partners sharing profits and losses equally, their original investments being $20,000, $3,000, and $45,000 respectively. Upon agreement to dissolve partnership, the debts are liquidated and the remaining assets converted into cash, realizing $22,000. How should the cash be divided? + +**Solution to Problem 18** + +Setting up a balance sheet, will give us the following: + + + + + + + + + + + + +
AssetsLiabilitiesEquity
$22,000 (cash)$18,750 (debts)$33,750 (equity)
+ +36 + +ILLUSTRATIVE PROBLEMS + +BALANCE SHEET OF BROWN, SMITH, AND JONES + + + + + + + + + + + + + + + + + + + + + + + + + + +
Cash$82,000.00Brown$99,000.00
Deficit48,000.00Smith5,000.00
Jones45,000.00
$79,000.00$79,000.00
+ +Dividing the deficit of $48,000 equally leaves Brown's account with a credit of $4,000, Smith's with a debit of $11,000, and Jones' with a credit of $29,000. + +Brown and Jones thus have a claim against Smith for $11,000, which when paid increases the cash to $83,000, this being shared by Brown and Jones in accordance with their capital interests: Brown $4,000; Jones $29,000. + +If the claim against Smith is uncollectible, there would be a loss of $11,000 to be borne equally by Brown and Jones, leaving Brown's capital account with a debit of $1,500, and Jones' with a credit balance of $23,500. Jones would then take the cash of $22,500 and have a claim against Brown for $1,500. + +Points Illustrated in Problem 18 + +(a) Distribution of Deficit and of Assets. As explained in Volume I, Chapter XXXII, § 6, the deficit should be distributed on the profit and loss sharing basis, whereas the cash should be distributed according to the amounts called for by the capital accounts. + +(b) Loss on Partner's Account. Where one partner has a debit balance which is uncollectible by the other partners, the latter share this loss in the ratio of their profit and loss sharing percentages. This matter is discussed in Volume I, Chapter XXXII, § 9. In this connection the partnership law (Chapter XXXIX of the Consolidated Laws), + +ILLUSTRATIVE PROBLEMS 37 + +passed in New York State on May 5, 1919, provides in Section 71, "If any, but not all, of the partners are insolvent, or, not being subject to process, refuse to contribute, the other partners shall contribute their share of the liabilities, and, in the relative proportions in which they share the profits, the additional amount necessary to pay the liabilities." + +**Problem 19** + +If the liquidation of a partnership consumes some time, it may be desired, in settlement, to pay off the partners in instalments as the assets are converted into cash. + +To illustrate the procedure in such a case, we will consider the case of a partnership consisting of R, G, and L, each with a credit to his capital account of $80,000. Profits and losses are divided, 50% to R, 30% to G, and 20% to L. The firm suffers a loss of $10,000, leaving assets of $50,000. The partners proceed to convert the assets into cash and later have cash to the amount of $20,000 to distribute. + +(a) How should the first instalment be divided? + +(b) The next instalment for distribution is $16,000 and the final instalment is $12,000. How should each of the last two instalments be divided? + +**Solution to Problem 19** + +(a) Dividing the loss of $10,000 among R, G, and L in the proportions stated above, leaves R with a + +38 + +ILLUSTRATIVE PROBLEMS + +capital balance of $15,000, G with $17,000, and L with $18,000. + +To make an equitable division of the first instalment, we should treat the unliquidated assets as a potential loss to be divided in proper proportions. This potential loss amounts to $80,000, the partners' shares being $15,000, $9,000, and $6,000 respectively. This leaves the capital accounts as follows: R none; G $8,000; L $12,000. On the first instalment, R therefore would receive nothing, G would receive $8,000, and L $12,000. R would then have a capital of $15,000, G $8,000, and L $6,000, which accounts now stand in the same ratio as that by which profits and losses are shared in accordance with the contract. + +(b) The second instalment of $16,000 would be shared in the proportion of 50, 30, and 20, R receiving $8,000, G $4,800, and L $3,200. The capital accounts would then show R $7,000, G $4,200, and L $2,800, or a total of $14,000. + +As the final instalment is only $12,900, a loss of $2,000 in liquidation has been sustained, which pro-rated leaves R with a capital of $6,900; C $3,600; and L $2,400. This is the basis on which the $12,900 would be divided. + +Points Illustrated in Problem 19 + +(a) Preventing Overpayment of Partner. The principal point in this problem concerns the method of distributing assets in partial payments upon dissolution of a firm in such a way as to prevent the overpayment of any one partner. This very important matter is fully discussed in Volume I, Chapter XXXII., § 8. + +ILLUSTRATIVE PROBLEMS 39 + +**Problem 20** + +Three partners contribute capital as follows: X $80,000, Y $45,000, Z $15,000. They share profits in the proportion of X 30%, Y 30%, and Z 20%. X's salary is $5,000, Y's salary is $3,000, Z's salary is $2,000. At the end of their fiscal period Z dies. The books are closed and the net assets ascertained to be $152,500, X and Y liquidate the firm's affairs and distribute the surplus assets quarterly as follows: + + + + + + + + + + + + + + +
First quarter$62,410.00
Second quarter74,622.30
Third quarter31,987.30
+ +$149,000.00 + +Prepare a statement of the partners' accounts, showing how the distribution of assets should be made, together with the apportionment of the loss. + +(NY C.P.A. Examination.) + +**Solution to Problem 20** + + + + + + + + + + + + + + +
Capital contributed$150,000.00
Salary credited to partners10,000.00
Accountability to partners160,000.00
+ +As there are net assets of only $152,500, there is a loss of $7,500 to be distributed: $3,750 to X, $8,250 to Y, $1,500 to Z. + +The assets are distributed in cash instalments as they are liquidated. In such a case, the basis of distribution must anticipate any shrinkage in assets in order that no partner shall be overpaid. The method is as follows: + +When the first instalment is ready for distribution, + +40 + +**ILLUSTRATIVE PROBLEMS** + +treat the remaining assets as a potential loss, and see what balances would remain to the credit of each partner if such a loss were to be divided in the proportion agreed upon. The credit balances resulting in such a case would be the basis for distribution of the first instalment. + +Proceeding in accordance with the method outlined we find that if the first instalment of $42,410.20 was all that would ever be realized from the assets, there would be a shrinkage of $11,069.80, which divided would show X's share to be $55,044.90, Y's $83,026.94, Z's $82,017.96. If these were charged against the partners, the resulting balances to the accounts would be: X, credit, $36,205.10; Y, credit, $12,723.06; and Z, debit, $6,517.96. + +Therefore, Z's debit must be treated as an additional potential loss to be borne by X and Y in the proportion of $\frac{2}{5}$ and $\frac{3}{5}$. X's share being $4,073.75$ and Y's $82,444.23$. Deducting these losses from the balances above, gives X a credit balance of $392,131.37$ and Y $810,278.83$, the basis for the division of the first instalment. + +This does not yet leave the accounts standing in the ratio of 30, 30, and 20, Z's account being proportionately less; since the purpose is to establish the above ratio as soon as possible, the second instalment is treated the same as the first. After it is ready for distribution the remaining assets of $35,467.50 are treated as a potential loss shared, X $17,733.75, Y $10,640.25, and Z $7,093.50, leaving X a credit balance of $41,384.88, Y $24,830.92, and Z $88,406.50, which is the basis for distribution of second instalment. + +A pie chart showing the distribution of assets among three partners. + +ILLUSTRATIVE PROBLEMS 41 + +The accounts of X, Y, and Z now show credit balances in the ratio of 50, 30, and 20, the same as the basis for sharing of profits. The third instalment is distributed in this proportion. As nothing then remains, there is a final loss in liquidation of $3,500, which is borne in the agreed ratio and in accordance with the capital accounts as they then stand. + +Following are skeleton ledger accounts with each partner. + + + + + + + + + + + + + + + + + + + + + + + + + + + + + + + + + + + + + + + + + + + + +
X
50% Net Loss$ 3,750.00Investment$90,000.00
First Instalment38,131.87Salary5,000.00
Second41,384.88
Third(30%)15,983.75
50% Net Loss1,256.09
$95,000.00
+ + + + + + + + + + + + + + + + + + + + + + + + + + + + + + + + + + + + + + + + + + + + +
Y
50% Net Loss$ 255.00Investment$45,000.00
First Instalment10,478.83Salary3,000.00
Second44,839.92
Third(30%)9,596.85
50% Net Loss1,165.09
$48,000.00
+ + + +
ZZ
+ + + +
Z
+ + + + +
Z + + + + +
Z + + + + +
Z + + + + +
Z + + + + +
Z + + + + +
Z + + + + +
Z + + + + +
Z + + + + +
Z + + + + +
Z + + + + +
Z + + + + +
Z + + + + +
Z + + + + +
Z + + + + +
Z + + + + +
Z + + + + +
Z + + + + +
Z + + + + +
Z + + + + +
Z + + + + +
Z + + + + +
Z + + + + +
Z + + + + +
Z + + + + + + + + + + + + + + + + + + + + + + + + + + + + + + + + + + + + + + + + + + + + + + + + + + + + + + + + + + + + + + + + + + + + + + + + + + + + + + + + + + + + + + + +
2/10 Net Loss$775.00Salary due
Balance






























































































Balance
$8259.69
                                               
Balance
$283.99
                  
Balance
$873.99
 
Balance
$923.99
 
Balance
$923.99
 
Balance
$923.99
 
Balance
$923.99
 
Balance
$923.99
 
Balance
$923.99
 
Balance
$923.99
 
Balance
$923.99
 
Balance
$923.99
 
Balance
$923.99
 
Balance
$923.99
 
Balance
$923.99
 
Balance
$923.99
 
Balance
$923.99
 
Balance
$923.99
 
Balance
$923.99
 
Balance
$923.99
 
Balance
+ +After crediting interest on capital and distributing the net loss, A's capital account shows a credit of + +44 +ILLUSTRATIVE PROBLEMS + +$8,812.50; B's a credit of $4,212.50; and C's a debit of $525. A and B should insist that the $250 salary due A be treated as a set-off against the debit balance in his account. This being the case, his remaining balance of $525 could be regarded as a potential loss to be shared by A and B in the proportion in which they share losses, the $12,500 cash then being divided between them in accordance with their adjusted capital accounts. If the claim of $825 against C could be collected, there would be $13,025 in cash with which to pay off the amounts due A and B. + +Points Illustrated in Problem 21 + +(a) Interest on Capital. The nature of interest on capital and the accounting means by which it is recorded are described in Volume I, Chapter XXXI, § 10. + +(b) Distribution of Loss and of Assets. As explained in Volume I, Chapter XXXII, § 6, the loss is distributed on the profit and loss account. The net cash is divided in accordance with the capital accounts after they have been adjusted. + +(c) Loss on Partner's Account. The distribution of such a loss among solvent partners is discussed in Volume I, Chapter XXXII, § 9. This point is covered also by the New York Partnership Law which is quoted in the points illustrated for Problem 18 to which reference should be made. + +(d) Salary of Partner. As explained in Volume I, Chapter XXXI, § 12, there can be, strictly speaking, no salary for any partner. The so-called salaries of partners which are chargeable against profits before the determination of the net amount divisible are discussed and explained in this reference. The New York Partnership Law passed May 3, 1919 (Chapter XXXIX of the Consolidated Laws) + +ILLUSTRATIVE PROBLEMS 45 + +provides in Section 40: "No partner is entitled to remuneration for acting in the partnership business." This, how-ever, does not prevent the allowance of the so-called sal-a-ry in the way explained in Volume I, Chapter XXXI, § 12. + +--- + +**Problem 22** + +**JONES AND ROSS** + +**TRIAL BALANCE** + +June 30, 1920 + + + + + + + + + + + + + + + + + + + + + + + + + + + + + + + + + + + + + + + + + + + + + + + + + + + + + + + + + + + + + + + + + + + + + + + + + + + + + + + + + + + + + + + + + + + + + + + + + + + +
Land (cost)8 $5,000.00
Building (cost)37,200.00
Furniture and Fixtures (cost)5,880.00
Cash7,682.53
Accounts Receivable65,751.40
Accounts Payable701.99
Inventory, December 31, 1919 (cost)$4,466.75
Mortgage Payable$ 35,000.00
Notes Payable9,840.03
Notes Payable5,000.00
C. R. Ross, Salary:-250.00
Reserve for Depreciation of Building :7,500.00
Reserve for Depreciation of Furniture and Fixtures:1,750.00
Reserve for Loss on Bad Accounts and Notes Re-cieve-able:109.89
H. B. Jones, Capital:-60,000.00
C. R. Ross, Drawings:1,869.00
C. R. Ross, Capital:-30,000.00
C. R. Ross, Drawings:4,763.05
Purchases:62,687.19
Freight, Express, and Cartage Inward:53,581.69
Traveling Expenses:1,984.54
Taxation Expenses:4,167.49
Delivery Expenses:3,866.72
Office Expenses:1,266.81
+ +C. R. Ross's share of net income for the year was $225. + + + + + + + + + + + + + + + + + + + + + + + + + + + + + + + + + + + + + +
46ILLUSTRATIVE PROBLEMS
Insurance$35.00
Interest on Notes Receivable136.84
Interest on Notes Payable$28.90
Interest on Accounts Payable$75.00
Cash Discounts on Purchases486.72
Cash Discounts on Sales372.04
$829,880.57
Cost of merchandise on hand June 30, 1920, $83,718.49.
+ +The firm of Jones and Ross conducts a wholesale and retail hardware business, owning its own real estate. + +By the terms of the partnership agreement, profits and losses are shared two-thirds by Mr. Jones and one-third by Mr. Ross: Mr. Ross who acts as general manager, is allowed a salary of $250 a month, which is considered as an expense of operating the business; profits not withdrawn by partners are not considered a part of their capital investments, but are credited to the partners' drawings accounts, and may be withdrawn by the partners at their convenience. On December 31, 1919, Mr. Jones' drawings account contained a credit balance of $3,629.40. Mr. Ross' drawings account had no balance. + +The item of freight, express, and cartage inward on merchandise purchases is not considered a part of the cost of goods purchased. The stock is very varied, and to distribute properly the cost of freight and carting among the numerous commodities would be difficult and unsatisfactory. + +During the six months ending June 30, 1920, the Sales account has been credited for $86,108.89 representing gross sales, and debited for $3,421.70 representing sales returns and allowances; the Purchases account + +ILLUSTRATIVE PROBLEMS 47 + +has been debited for $57,529.46 gross purchases, and credited for $4,207.86 purchase returns and allowances. + +In order that the results of the period may be correctly shown, the following items require adjustment: + + + + + + + + + + + + + + + + + + + + + + +
Unexpired insurance as of June 30$60.00
Taxes accrued to June 30102.50
Interest accrued on interest-bearing notes receivable to June 3084.60
Interest accrued on interest-bearing notes payable to June 3075.00
Office supplies on hand which cost150.89
+ +Depreciation on the building is figured at the rate of 2% per annum; on the furniture and fixtures, at 10% per annum. It is desired to set aside out of the profits for the period a further reserve for loss on bad accounts and notes receivable amounting to ½% of the net sales. + +Required: +(a) Adjusting entries +(b) Working sheet +(c) Profit and loss statement +(d) Balance sheet +(e) Closing entries + +**Solution to Problem 22** + +**(a)** + +JONES AND ROSS + +ADJUSTING ENTRIES +June 30, 1920 + + + + + + + + + + +
Unexpired Insurance$60.00
Insurance$60.00
+ +To bring onto the books the unexpired insurance as of this date. + + + + + + + + + + + + + + + + + + + + + + + + + + + + + + + + + + + + + + + + + + + + + + + + + + + + + + + + + + + + + + + + + + + + + + + + + + + + + + + + + + + + + + + +
48 ILLUSTRATIVE PROBLEMS
Taxes102.50
Taxes Accrued102.50
To bring onto the books the taxes accrued to date.
Accrued Interest on Notes Receivable84.60
Interest on Notes Receivable84.60
To bring onto the books the interest accrued to date on interest-bearing notes receivable:
J. A. Shore's note of October 15, 1919, $500,
8 months, 1% per month, at 6%841.25
R. C. O'Neal's note of June 8, 1920, $500, 6
months, 27 days at 6%3.35
Interest on Notes Payable75.00
Interest Accrued on Notes Payable75.00
To bring onto the books the interest accrued to date on interest-bearing notes payable:
Note of April 1, 1920, favor First National Bank,
$5,000, 90 days at 6%
Office Supplies on Hand150.89
Other Supplies on Hand150.89
To bring onto the books the cost of office supplies on hand as of this date.
Depreciation of Building375.00
Reserve for Depreciation of Building375.00
Estimated depreciation of building for the six months ending June 30, 1920. Figured on cost (875,500) at the rate of $2% per annum.
+ + + + + + + + + + + + + + + + + + + + + + + + + + + + + + + + + + + + + + + + + + + + + + + + + + + + +ILLUSTRATIVE PROBLEMS +49 + +(c) +JONES AND ROSS +TRADING AND PROFIT AND LOSS STATEMENT +For Six Months Ending June 30, 1920 + +Net Sales: +Gross Sales ..... $866,108.89 +Less Returns and Allowances ..... 5,411.70 $88,697.19 +Deduct—Cost of Sales ..... $44,260.75 +Goods on Hand January 1, 1920 ..... $44,260.75 +Net Purchases: +Gross Purchases ..... 857,529.46 +Less Returns and Allowances ..... 4,407.86 +$532,160 $77,582.55 +Less—Goods on Hand June 30, 1920 ..... $21,710.40 $31,571.95 +Gross Trading Profit ..... $830,815.94 + +Deduct—Operating Expenses: +Freight, Express, and Cartage Forwarded ..... $1,993.34 +Express and Telegrams ..... $1,070.40 +Salaries and Wages ..... 5,300.78 +Delivery Expenses ..... 1,100.81 +Other Operating Expenses ..... 14,966.21 +Taxes ..... 106.50 +Insurance ..... 173.00 +Depreciation of Buildings ..... 372.00 +Depreciation of Furniture and Fixtures ..... 831.90 17,071.98 +Net Trading Profit ..... $815,743.82 + +Add—Extraneous Income Items: +Interest on Notes Receivable ..... 8 160.84 +Cash Discounts on Purchases ..... 498.72 647.56 +Total Income ..... $814,390.82 + +Deduct—Extraneous Expense Items: +Interest on Notes Payable ..... $315.90 +Interest on Mortgage Payable ..... 873.00 +Cash Discounts on Sales ..... 372.02 +Loss on Bad Debts and Accounts Receivable ..... 415.44 1,974.56 +Net Profit for Period: R H Jones-two-thirds ..... $8,877.64 +C H Ross-one-third ..... -4,138.86 $4,738.82 + +
+ + + + + + + + + + + + + + + + + + + + + + + + + + + + + + + + + + + + + + + + + + + + + + + + + + + + + + + + + + + + + + + + + + + + + + + + + + + + + + + + + + + + + + + + + + + + + + + + + + + + + + + + + + + + + + + + + + + + + + + + + + + + + + + + + + + + + + + + + + + + + + + + + + + + + + + +
$814,390.82
$814,390.82
R H Jones-two-thirds$8,877.64$814,316.46
+
C H Ross-one-third








































































+
ACCOUNTSTOTAL BALANCE FOR BOOKSARGUMENTS
DebitsCreditsDebitsCredits
1. Land...$ 34,000.00
2. Building...22,000.00
3. Furniture and Fixtures...3,850.00
4. Accounts Receivable...6,771.40
5. Notes Receivable...1,850.00
6. Inventory...84,998.72$15,000.00
7. Merchandise Putable...
8. Accounts Payable...9,245.00
9. Notes Payable...1,850.00
DescriptionAmount
Reserve for Depreciation of Building
Reserve for Depreciation of Furniture and Fixtures
Reserve for Less on Bad Accounts and Notes Receivable
H. B. Jones, Drawings
C. R. Jones, Drawings
Furniture and Fixtures
Merchandise Putable
Accounts Payable
Notes Payable
Reserve for Depreciation of Building
Reserve for Depreciation of Furniture and Fixtures
Reserve for Less on Bad Accounts and Notes Receivable
H. B. Jones, Drawings
C. R. Jones, Drawings
Furniture and Fixtures
Merchandise Putable
Accounts Payable
Notes Payable
Reserve for Depreciation of Building
Reserve for Depreciation of Furniture and Fixtures
Reserve for Less on Bad Accounts and Notes Receivable
H. B. Jones, Drawings
C. R. Jones, Drawings
Furniture and Fixtures
Merchandise Putable
Accounts Payable
Notes Payable
Reserve for Depreciation of Building
Reserve for Depreciation of Furniture and Fixtures
Reserve for Less on Bad Accounts and Notes Receivable
H. B. Jones, Drawings
C. R. Jones, Drawings
Furniture and Fixtures
Merchandise Putable
+ + +
1.
+ +
Land...
+ +
Building...
+ +
Furniture and Fixtures...
+ +
Accounts Receivable...
+ +
Notes Receivable...
+ +
Inventory...
+ +
Merchandise Putable...
+ +
Accounts Payable...
+ +
Notes Payable...
+ +
Reserve for Depreciation of Building...
+ +
Reserve for Depreciation of Furniture and Fixtures...
+ +
Reserve for Less on Bad Accounts and Notes Receivable...
+ +
H. B. Jones, Drawings...
+ +
C. R. Jones, Drawings...
+ +
Furniture and Fixtures...
+ +
Merchandise Putable...
+ +
Accounts Payable...
+ +
Notes Payable...
+ +
Reserve for Depreciation of Building...
+ +
Reserve for Depreciation of Furniture and Fixtures...
+ +
Reserve for Less on Bad Accounts and Notes Receivable...
+ +
H. B. Jones, Drawings...
+ +
C. R. Jones, Drawings...
+ +
Furniture and Fixtures...
+ +
Merchandise Putable...
+ + +Accounts Payable... + + +Notes Payable... + + +Reserve for Depreciation of Building... + + +Reserve for Depreciation of Furniture and Fixtures... + + +Reserve for Less on Bad Accounts and Notes Receivable... + + +H. B. Jones, Drawings... + + +C. R. Jones, Drawings... + + +Furniture and Fixtures... + + +Merchandise Putable... + + +Accounts Payable... + + +Notes Payable... + + +Reserve for Depreciation of Building... + + +Reserve for Depreciation of Furniture and Fixtures... + + +Reserve for Less on Bad Accounts and Notes Receivable... + + +H. B. Jones, Drawings... + + +C. R. Jones, Drawings... + + +Furniture and Fixtures... + + +Merchandise Putable... + + +Accounts Payable... + + +Notes Payable... + + +Reserve for Depreciation of Building... + + +Reserve for Depreciation of Furniture and Fixtures... + + +Reserve for Less on Bad Accounts and Notes Receivable... + + +H. B. Jones, Drawings... + + +C. R. Jones, Drawings... + + +Furniture and Fixtures... + + +Merchandise Putable... + +ILLUSTRATIVE PROBLEMS +51 + +AND ROSS +SHEET +1920, to June 30, 1920 + + + + + + + + + + + + + + + + + + + + + + + + + + + + + + + + + + + + + + + + + + + + + + + + + + + + + + + + + + + + + + + + + + + + + + + + + + + + + + + + + + + + + + + + + + + + + + + + + +
DebitsCreditsTradingProfit and LossAssets and Liabilities
1$ 2,000.00$ 2,000.00
2$ 7,806.00$ 7,806.00
3$ 7,806.00$ 7,806.00
4$ 7,806.00$ 7,806.00
5$ 7,806.00$ 7,806.00
6$ 7,806.00$ 7,806.00
7$ 7,806.00$ 7,806.00
8$ 7,806.00$ 7,806.00
9$ 5,495.45$ 5,495.45
10$ 5,495.45$ 5,495.45
11$ 5,495.45
$ 2,394.31$ 3,121.14$ 2,394.31$ 3,121.14$ 2,394.31$ 3,121.14
$ 2,394.31
$ 3,121.14
$ 2,394.31
$ 3,121.14
$ 2,394.31
$ 3,121.14
$ 2,394.31
$ 3,121.14
$ 2,394.31
$ 3,121.14
$ 2,394.31
$ 3,121.14

+ $ 5,495.45 + $ 7,806.00 + $ 5,495.45 + $ 7,806.00 + $ 5,495.45 + $ 7,806.00 + $ 5,495.45 + $ 7,806.00 + $ 5,495.45 + $ 7,806.00 + $ 5,495.45 + $ 7,806.00 + $ 5,495.45 + $ 7,806.00 + $ 5,495.45 + $ 7,806.00 + $ 5,495.45 + $ 7,806.00 + $ 5,495.45 + $ 7,806.00 + $ 5,495.45 + $ 7,806.00 + $ 5,495.45 + $ 7,806.00 + $ 5,495.45 + $ 7,806.00 + $ 5,495.45 + $ 7,806.00 + $ 5,495.45 + $ 7,806.00 + $ 5,495.45 + $ 7,806.00 + $ 5,495.45 + $ 7,806.00 + $ 5,495.45 + $ 7,806.00 + $ 5,495.45 + $ 7,806.00 + $ + +52 + +**ILLUSTRATIVE PROBLEMS** + +**(d)** + +**JONES AND ROSS** + +**BALANCE SHEET** + +**June 30, 1920** + +**Assets** + + + + + + + + + + + + + + + + + + + + + + + + + + + + + + + + + + + + + + + + + + + + + + + + + + + + + + + + + + + + + + + + + + + + + + + + + + + + + + + + + + + + + + + + +
Fixed Assets:
Land (cost)$87,500.00$55,000.00
Building (cost)
Less—Reserve for Depreciation7,875.009,025.00
Furniture and Fixtures (cost)5,820.60
Less—Reserve for Depreciation2,941.003,779.00
$8,404.00
Current Assets:
Cash on Hand$7,682.55
Accounts Receivable823,731.40
Notes Receivable730.00
Less—Reserve for Bad Debts and Notes Receivable583.84$3,878.16
Accrued Interest on Notes Receivable$4.60
Merchandise on Hand83,710.4057,495.69
Deducted Charges to Profit and Loss:
Unexpired Insurance$650.00
Cross Supplies on Hand150.89
Total Assets$146,110.58
+ +**Liabilities and Net Worth** + + + +
Fixed Liabilities:
+ + + +
Mortgage Payable
+ + + +
Mortgage Payable $85,000.00
+ + + +
Current Liabilities:
+ + + +
Accounts Payable $9,840.62
+ + + +
Notes Payable $3,000.00
+ + + +
Accrued Interest on Notes Payable $75.00 + +ILLUSTRATIVE PROBLEMS +53 + +Acerued Taxes 102.39 +Duc C. R. Ross on Salary Account 250.00 +Total Liabilities 8 50,608.12 + +H. B. Jones' Net Worth: +Capital Investment $60,000.00 +Add: +Profits Accumulated to January 1, 1920 8 3,049.40 +Two-thirds Net Profit for six months ending June 30, 1920 8,277.64 +Less--Drawings, January 1, 1920, to June 30, 1920 811,997.04 +$66,408.64 +C. B. Ross' Net Worth: +Capital Investment $80,000.00 +Debts due to Jones, January 1, 1920, to June 30, 1920 8 4,755.00 +Less--One-third Net Profit for six months ending June 30, 1920 4,138.82 +566.18 +$9,493.82 +Total Liabilities and Net Worth $146,110.58 + +(e) + +JONES AND ROSS +CLOSING ENTRIES +June 30, 1920 + +Trading Inventory $24,890.75 +Goods on hand January 1, 1920, per inventory $24,890.75 +Trading Purchases 53,321.60 +Net purchases for six months ending June 30, 1920 53,321.60 +Sales Trading $82,687.19 +Net sales for six months ending June 30, 1920 $82,687.19 + +54 + +**ILLUSTRATIVE PROBLEMS** + + + + + + + + + + + + + + + + + + + + + + + + + + + + + + + + + + + + + + + + + + + + + + + + + + + + + + + + + + + + + + + + + + + + + + + + + + + + + + + + + + + +
Inventory$3,710.40
Trading$3,710.40
Goods on hand June 30, 1920, per inventory.
Trading30,815.24
Profit and Loss30,815.24
To transfer Profit and Loss account the gross trading profit for the six months ending June 30, 1920, as represented by the balance of the Trading account.
Interest on Notes Receivable160.84
Cash Discounts on Purchases486.72
Profit and Loss647.56
To transfer to Profit and Loss account the bal- + ances of the accounts representing extraneous + income for the six months ending June 30, 1920.
Profit and Loss17,071.88
Freight, Express, and Cartage Inward1,093.54
Traveling Expenses4,107.40
Salesmen's Salaries9,869.81
Delivery Expenses1,390.81
Office Expenses1,436.21
Taxes:192.50
Insurance175.00
Depreciation of Building375.00
Depreciation of Furniture and Fixtures491.00
To transfer to Profit and Loss account the bal- + ances of the accounts representing operating + expenses for the six months ending June 30, + 1920.Profit and Loss1,974.36
Interest on Notes Payable +                                                                                                                 &nbsp; < < < < < < < < < < < < < < < < < < < < < < < < < < < < < < < < < < < < < < < < < < < < < < < < < + + + + ++ + + + + + + + + + + + + + + + + + + + + + + + + + + + + + + + + + + + + + + + + + +
Cash8 $5,000.00Accounts Payable$25,000.00
Accounts Receivable30,000.00Notes Payable5,000.00
Investments175,000.00Sundry Assets$60,000.00
Sundry Assets$25,000.00Jones, Capital$60,000.00
Clark, Capital$30,000.00
$185,000.00
$185,000.00
+ +They incorporate "The Smith-Jones Company" with an authorized capital stock of $175,000, all the + +ILLUSTRATIVE PROBLEMS 57 + +stock to be issued to the three partners in exchange for their respective interests in the business. + +Closing entries for the partnership books and opening entries for the corporation are required. + +**Solution to Problem 23** + +**ENTRIES TO CLOSE PARTNERSHIP BOOKS** + + + + + + + + + + + + + + + + + + +
Good-Will$ 25,000.00
Smith, Capital$ 10,000.00
Jones, Capital$ 10,000.00
Clark, Capital$ 5,000.00
+ +To bring the accounts to the partnership books, +its value being determined by the excess of the capital stock to be issued to the partners over the net worth of their holdings as shown by the balance sheet dated December 1. +Good-Will is divided among them in the proportion in which they share profits. + +The Smith-Jones Company + + + + + + + + + + + + + + + + + + + + + + +
Cash$ 25,000.00
Accounts Receivable$ 5,000.00
Inventories$ 18,000.00
Sundry Assets$ 25,000.00
Good-Will$ 25,000.00
+ +To record the transfer of assets taken over by The Smith-Jones Company. + + + + + + + + + + +
Accounts Payable$ 25,000.00
Notes Payable$ 3,000.00
+ +To record the transfer of liabilities assumed by The Smith-Jones Company. + + + + + + +
The Smith-Jones Company$ 30,000.00
+ +Capital Stock (The Smith-Jones Company) + + + + + + +
The Smith-Jones Company$ 175,000.00
+ +To bring onto the books the $175 shares of stock received from The Smith-Jones Company in exchange for the partnership business. + +58 + +**ILLUSTRATIVE PROBLEMS** + +Smith, Capital .................................................. $70,000.00 +Jones, Capital .................................................... $70,000.00 +Clark, Capital ..................................................... $33,000.00 +Capital Stock ...................................................... 175,000.00 + +To show the interest to the individual partners of the capital stock of The Smith-Jones Company, thus closing all the accounts on partnership books. + +**ENTRIES TO OPEN CORPORATION BOOKS** + +December 1, 1919 + +The Smith-Jones Company has been incorporated this day under the laws of the State of Massachusetts, with an authorized capital stock of $175,000, divided into 1,750 shares of the par value of $100 each. + + + + + + + + + + + + + + + + + + + + + + + + + + + + + + + + + + + + + + + + + + + + + + + + + + + + + + +
Cash$ 8,000.00
Accounts Receivable50,000.00
Inventories65,000.00
Sundry Assets25,000.00
Good-Will25,000.00
Smith, Jones and Company$855,000.00
To bring onto the books the assets acquired from Smith, Jones and Company.
Smith, Jones and Company95,000.00
Accounts Payable25,000.00
Notes Payable5,000.00
To bring onto the books the liabilities of Smith, Jones and Company assumed by the corporation.
Smith, Jones and Company - Capital Stock175,000.00
175,000.00
+ +To show the issue to Smith, Jones and Company of 1,750 shares of stock in exchange for the business formerly conducted by them as a partnership. + +**Points Illustrated in Problem 23** + +(a) Good-Will on Dissolution. As explained in Volume I, Chapter XXXII, § 10, a partnership upon its dissolution + +ILLUSTRATIVE PROBLEMS 59 + +under circumstances such as those given in this problem may record a good-will upon its books. Such good-will constitutes a profit and is divisible among partners in the profit and loss ratio. + +(b) Closing Partnership Accounts. The customary procedure involved in closing the accounts of a partnership upon its dissolution is fully described in Volume I, Chapter XXXII, § 10. + +(c) Opening Entry for Corporation. The entry required to open corporation books when good-will is purchased under conditions similar to those in this problem is discussed in Volume I, Chapter XXXIII, § 8. + +**Problem 24** + +Ralph H. Wilson, George D. Light, and Frank L. Spear organize the R. H. Wilson Company with an authorized capital stock of $50,000. The subscription books close June 1, all the stock having been subscribed for. The subscribers pay their subscriptions in cash July 1, and receive stock certificates covering their holdings. + +Show by pro forma journal entries how the books would be opened. + +**Solution to Problem 24** + +June 1 + + + + + + + + + + +
Subscriptions Receivable$30,000.00
Capital Stock Subscribed$50,000.00
+ +The full authorized issue of stock of the R. H. Wilson Company has been subscribed for, subscriptions payable July 1. + +60 +ILLUSTRATIVE PROBLEMS + +July 1 + + + + + + + + + + + + + + + + + + + + + + + + + + +
Cash50,000.00
Subscriptions Receivable50,000.00
Cash received in payment of subscriptions.
Capital Stock Subscribed50,000.00
Capital Stock50,000.00
Stock certificates issued to the stockholders.
+ +**Points Illustrated in Problem 24** + +(a) Subscriptions to Capital Stock. In this problem the capital stock is not immediately issued for cash or other considera- +tion but is subscribed for by various individuals. The +bookkeeping necessary under these conditions is outlined +in Volume I, Chapter XXXIII, § 6, and in Volume IV, +Chapter XIV, § 4. + +**Problem 25** + +Following is the liabilities and capital section of the +balance sheet of a corporation: + + + + + + + + + + + + + + + + + + + + + + + + + + + + + + + + + + + + + + + + + + + + + + + + + + + + + + + + + + + + + + + +  
Capital Liabilities:
Preferred Stock$100,000.00
Common Stock50,000.00
First Mortgage Bonds100,000.00
$700,000.00
Current Liabilities:
Notes Payable8 $40,000.00
Acceptances Payable12,000.00
Accounts Payable18,999.99
170,000.09
Reserves:
Depreciation8 $20,000.09
Bad Debts5,333.33
Insurance18,999.99
Costs and Contingencies53,333.33
Total$166,415.89$166,415.89$166,415.89$166,415.89$166,415.89$166,415.89$166,415.89$166,415.89$166,415.89$166,415.89$166,415.89$166,415.89$166,415.89$166,415.89$166,415.89$166,415.89$166,415.89$166,415.89$166,415.89$166,415.89$166,415.89$166,415.89$166,415.89$166,415.89$166,415.89$166,415.89$166,415.89$166,415.89$166,415.89$166,415.89$166,415.89$166,415.89$166,415.89$166,415.89$166,415.89$166,415.89$166,415.89$166,415.89$166,415.89$166,415.89$ + +ILLUSTRATIVE PROBLEMS 61 + +Calculate the book value of the common stock (par value $50 per share). + +**Solution to Problem 25** + + + + + + + + + + + + + + + + + + + + + + + + + + + + + + + + + + +
Net Worth of Company:
Preferred Stock$100,000.00
Common Stock500,000.00
Surplus136,418.20
Insurance Reserve10,000.00
Contingent Reserve80,000.00
Deduct—Preferred Stock Equity100,000.00
Common Stock Equity$666,418.20
+ +$666,418.20 divided by 10,000 shares equals $66.64 book value of common stock. + +**Points Illustrated in Problem 25** + +(a) Preferred Stock. The nature of preferred stock is explained in Volume I, Chapter XXXIII, § 5. + +--- + +**Problem 26** + +The Nassau Engineering Company fails and a receiver is appointed on March 1, 1920, who on taking charge finds the company's liabilities and assets to be as follows: creditors, unsecured $59,100, partly secured $16,500, fully secured $13,500. The company owns real estate $15,000, which is mortgaged for $10,000; machinery and tools $30,000; materials $3,000, and book debts $9,000, including $2,500 in litigation on which a loss of 50% is expected; also securities of the + +62 + +ILLUSTRATIVE PROBLEMS + +value of $22,500 acquired in settlements, of which $7,500 are pledged with partly secured creditors and $14,000 with fully secured creditors. There are engineering contracts in force to the amount of $60,000, on which $45,000 has been expended. Cash in bank $750. The capital stock of the company is $75,000, and the accumulated losses on contracts, bad debts written off, and expenses show a deficiency of $48,850. Customers' bills have been discounted to the amount of $4,500, of which $1,500 will be dishonored in consequence of failure of obligor. The machinery and tools are expected to realize only 50% of the book value, and the real estate is appraised at $12,000. The cost to complete contracts is estimated at $30,000 by the sureties who offer $9,230 for the stock of materials on hand. Unpaid taxes and assessments amounting to $216 are undiscovered but no entry thereof appears in the company's books. + +Prepare a statement of affairs and deficiency account in technical form. + +**Solution to Problem 26** + +NASSAU ENGINEERING COMPANY + +STATEMENT OF AFFAIRS + +March 1, 1920 + + + + + + + + + + + + + + + + + + + + +
Book ValueAssetsExpected to RealizeDeficiency
8$750.00 Cash$750.00$750.00
9,000.00Accounts Receivable$4,500 in liquidation on which a loss of 59% is expected.$4,500
+ +ILLUSTRATIVE PROBLEMS 63 + + + + + + + + + + + + + + + + + + + + + + + + + + + + + + + + + + + + + + + + + + + + + + + + + + + + + + + + + + + + + + + + + + + + + + + + +
Book ValueExpected to RealizeDeficiency
$25,000Securities Owned1,500.00
$14,000 pledged with $11,000 secured creditors having claims of $815,500.00 Pledged with partly se- cured creditors$81,000.00
Materials$2,250.00
Engineering Contracts Contract Price Less—Estimated cost to complete30,000.00
$60,000.0015,000.00
Less—Estimated cost to complete$30,000.00
Real Estate Appraised Value Lease—Mortgage (per contra)$2,000.00
$12,000.003,500.00
Lease—Mortgage (per contra)$12,000.00
Totals$35,850.00
$125,250.00Machinery and Tools Balance available for Un- secured Creditors (Indi- cating amount required to creditors of approxi- mately 83%) Deficiency per Deficiency Account$35,850.00
Deduct—Preferred Claims (per contra): Unpaid Taxes and As- sessments$16.66
Total Deficiency per Deficiency Account$69,659.66
+ +64 + +# ILLUSTRATIVE PROBLEMS + +## Liabilities + + + + + + + + + + + + + + + + + + + + + + + + + + + + + + + + + + + + + + + + + + + + + + + + + + + + + + +
Book ValueExpected to Rank
$ 89,100.00$69,100.00
Accounts Payable
Deducted contra$41,000.00
10,000.00
Materials
Deducted contra10,000.00
Notes Receivable - Discounted
Counted1,500.00
Unpaid Assessments
Mortgage Payments
Deducted contra216.00
Total$69,600.00
+ + + +
Capital Obligations:
Capital Stock$75,000.00
Less-Deficit48,850.00
$6,150.00Net Worth $86,150.00
$125,450.00
+ +NASSAU ENGINEERING COMPANY +DEFICIENCY ACCOUNT +March 1, 1929 + + +
Loss on Accounts ReceivableCapital Stock$75,000.00
$ 1,235.00Deficiency per State Department of Affairs10,566.00
Loss on Materials756.88
Loss on Uncompleted Contracts15,000.00
Loses on Equipment and Machinery and Tools5,996.88
Loses Not on Books:15,996.88
Taxes and Assessments:- $16.00
Loss on Discounted Note:1,506.88
Debt Balance of Profit and Loss Account:$45,836.88$85,566.88
+ +ILLUSTRATIVE PROBLEMS +65 + +Points Illustrated in Problem 26 + +(a) Statement of Affairs. The reasons for the technical and special form of this statement are given in Volume I, Chapter XII, § 9. + +(b) Differences from Balance Sheet. The points in which a statement of affairs differs from a balance sheet are described in Volume I, Chapter XII, § 3. The principal differences are in the valuation of assets and in the deductions of assets from liabilities, or vice versa, to record the securities held by creditors. + +(c) Deficiency Account. This account is in the account form but it might be presented equally well in the statement or running form. The purpose of the account is described in Volume I, Chapter XII, § 11. + +(d) Preferred Claims. The nature of preferred claims and the method of stating them in the statement of affairs are discussed in Volume I, Chapter XII, § 6, and in Volume IV, Chapter XXVII, § 1. + +(e) Expected Dividends. This solution indicates that creditors may reasonably expect a dividend of approximately 85%. In other words, each creditor is likely to receive 85 cents on the dollar. The practicability of showing this information is discussed in Volume I, Chapter XII, § 10. + +(f) Notes Receivable Discounted. This is a contingent liability which is expected to become an actual one. For a discussion of its kind see Liability, importance of indicating it, see Volume I, Chapter XXVII, § 1. + +(g) Distribution of Loss. The deficiency account shows that the total loss is borne in part by the owners and in part by the creditors. The portion borne by the latter is $10,566$, as shown by the statement of affairs. For a discussion of this function of the deficiency account, see Volume I, Chapter XII, § 10. + +(h) Net Worth. On the statement of affairs the deficit is deducted from the capital stock to secure the net worth. This procedure is discussed in Volume IV, Chapter XXI. + +A page from a book with text discussing illustrative problems. + +66 +ILLUSTRATIVE PROBLEMS + +§7. Another reason for deducting this from the capital stock is to distinguish between the operating loss which resulted in the deficit and the loss on the realization of the assets. + +--- + +**Problem 27** + +The following is a trial balance of the books of the X Y Z Manufacturing Company, which has been declared bankrupt: + +TRIAL BALANCE +At June 30, 1914 + + + + + + + + + + + + + + + + + + + + + + + + + + + + + + + + + + + + + + + + + + + + + + + + + + + + + + + + + + + + + + + + + +
Real Estate and Buildings$185,000.00
Capital Stock$890,000.00
Machinery and Equipment160,000.00
Customers' Accounts Receivable170,000.00
Notes Payable$50,000.00
Accounts Payable512,000.00
Insurance Premiums Unexpired3,000.00
Mortgage on Buildings65,000.00
Notes Payable on Mortgage26,000.00
Interest Accrued on Mortgage8,500.00
Cash on Hand and in Bank6,500.00
Inventory of Raw Material85,000.00
Inventory of Finished Goods185,000.00
Investment18,500.00
Deficit$21,950.00
$293,550.00$293,550.00
+ +The real estate and buildings are appraised at $119,167. The machinery and equipment at $185,352. +An examination of the customers' accounts shows the following condition: good $895,167, doubtful (expect to collect $31%$) $851,167, bad $24,244$. The holders of the notes payable of $18,244 hold notes receivable in $24,244. + +ILLUSTRATIVE PROBLEMS 67 + +security of face value of $15,000, but worth only $10,000. +A creditor of $55,000 on open account has in his possession the stock certificates for the investments assigned in blank and finished goods pledged to the value of $16,000. The insurance premiums unexpired have a cash value of $2,200. An examination of the notes receivable shows $9,000 good for collection, and $17,000 doubtful on which 50% will be collected. The investments have a marketable value of $16,300. + +Prepare statement of affairs for submission to creditors, showing the amount on the dollar the creditors may expect to receive; also prepare deficiency statement. + +(Missouri C. P. A. Examination.) + +**Solution to Problem 27** + +THE X. Y. Z. MANUFACTURING COMPANY + +STATEMENT OF AFFAIRS + +At June 30, 1914 + + + + + + + + + + + + + + + + + + + + + + + + + + + + + + + + + + + + + + + + + + + + + + + + + + + + + + + + + + + + + + + + + + + +
AssetsBook ValueEstimated to Realize
Cash on Hand and in Bank8,650.008,650.00
Accounts Receivable:
Good93,000.0093,000.00
Doubtful51,000.0017,000.00
Bad44,000.00
Notes Receivable:
Good9,998.889,998.88
Doubtful17,500.008,500.00
Less—Notes held to secure Notes Payable:$17,500.00
Book Value$15,000.00
Will Realize10,000.0010,000.00
7,500.00
+ + + + + + + + + + + + + + + + + + + + + + + + + + + + + + + + + + + + + + + + + + + + + + + + + + + + + + + + + + + + + + + + + + + + +
68ILLUSTRATIVE PROBLEMSEstimated to Realize
Book Value
Finished Goods$12,000.00$12,000.00
Less—Pledged to secure Account Payable$16,000.00
Investments$12,000.00$16,500.00$16,500.00
Less—Pledged to secure Account Payable$16,500.00
Inventory Raw Material85,000.00$3,600.00
Insurance Premium Unexpired5,000.00$2,800.00
Materials in Transit181,500.00131,939.99
Real Estate and Buildings123,499.99$101,957.57
Less—Mortgage and Interest$67,558.58$35,558.58
Deficit$21,000.00$929,558.58
Total$929,558.58
Amount available for distribution to Unsecured creditors and tenant 93.68% of their claims, subject to realization and liquidation expense$486,767.67
Deficiency, per Deficiency Account$31,781.44
Liabilities
Preferred Claims (none)Book ValueEstimated to Rank
Fully Secured Claims:
Mortgage on Buildings$63,558.58
Interest on Mortgage$2,846.77
Deducted from Assets (per contra)
Partly Secured Claims:
Note Payable$25,846.77
Less—Notes Receivable (de-ducted per contra)$16,558.58$246,767.67
Accounts Payable$246,767.67
Less:
+ +
Finished Goods Pledged (per contra)
+ + + +
Investments Pledged (per contra)
+ + + +
$16,558.58
+ + + +
$246,767.67
+ + + +
$246,767.67
+ + + +
$246,767.67
+ + + +
$246,767.67
+ + + +
$246,767.67
+ + + +
$246,767.67
+ + + +
$246,767.67
+ + + +
$246,767.67
+ + + +
$246,767.67
+ + + +
$246,767.67
+ + + +
$246,767.67
+ + + +
$246,767.67
+ + + +
$246,767.67
+ + + +
$246,767.67
+ + + +
$246,767.67
+ + + +
$246,767.67
+ + + +
$246,767.67
+ + + +
$246,767.67
+ + + +
$246,767.67
+ + + +
$246,767.67
+ + + +
$246,767.67
+ + + +
$246,767.67
+ + + +
$246,767.67
+ + + +
$246,767.67
+ + + +
$246,767.6769 + + + + + + + + + + + + + + + + + + + + + + + + + + + +
Unsecured Claims:Book ValueEstimated to Rank
Accounts Payable$37,000.00$37,000.00
Capital Stock300,000.00
Total8929,500.00
Total Unsecured Claims8519,500.00
+ +DEFICIENCY ACCOUNT + + + + + + + + + + + + + + + + + + + + + + + + + + + + + + + + + + + + + + + + + + + + +
Book Deficit$221,000.00Capital Stock8509,000.00
Shrinkage in Accounts Receivable38,000.00Appreciation of Investments4,300.00
Shrinkage in Insurance Premium Unexpired.899.00Net Deficiency, per Statement of Affairs92,800.00
Shrinkage in Notes Receivable ...8,500.00
Shrinkage in Machinery and Equipment.25,959.00
Shrinkage in Real Estate.24,568.00
amendment.
$837,599.00
+ +Points Illustrated in Problem 27 + +(a) Condensed Form. This form of statement of affairs differs from that in Problem 26 in that the shrinkage of each asset is not indicated in the statement. Such losses on realization are shown only in the deficiency account. Reference should be made to Problem 26 for a more complete statement. + +(b) Ambiguity in Problem. There is an ambiguity in the problem in the statement, "A creditor of $35,599 on open account has in his possession the stock certificates for the investments assigned in blank and finished goods pledged to the value of $16,599." From this statement one may assume either that the creditor holds a total security of + +70 + +ILLUSTRATIVE PROBLEMS + +$16,000 consisting of $12,000 in securities and $4,000 in finished goods, or that he holds $12,000 in securities and $16,000 in finished goods, making his total $28,000. The above solution is based on the second assumption. It is to be regretted that state boards of examiners do not avoid ambiguities of this sort. + +(c) Interest on Mortgage. The solution properly shows that interest on the mortgage is secured in the same way that the principal is secured. In other words, when a mortgage loan is secured, the security applies to both principal and interest. + +(d) Unexpired Insurance. The problem states that unexpired insurance has a cash value of $2,200. Accordingly, the solution states this amount in the "Estimated to Realize" account. It is questionable whether this treatment is correct. If the problem were less definite in its statement of the cash value, the unexpired insurance would be taken to have no realizable value because it would be assumed that the insurance must be continued. + +(e) Appreciation of Assets. The assets which have a book value of $12,000 but are expected to realize $16,500. This gain on realization would, of course, be an offset to the loss and would constitute an increase in the capital of the business. + +--- + +Problem 28 + +The firm of Smith and Jones, of Columbus, Ohio, a partnership, was forced into bankruptcy. To protect creditors, each of the partners has also filed a voluntary petition in bankruptcy. + +A trial balance taken from the firm books at October 31, 1913, is as follows: + + + + + + +
+ +ILLUSTRATIVE PROBLEMS 71 + +John Smith 8 $35,000.00 +William Jones 45,000.00 + +Cash 8 $75.00 +Land 16,000.00 +Buildings 39,500.00 +Machinery, Tools, etc. 33,500.00 +Furniture and Fixtures 1,500.00 +Horses and Wagons 2,550.00 +Accounts Receivable 58,900.00 +Notes Payable 18,790.00 + +Mortgage Payable—Real Estate 25,660.00 +Accounts Payable 163,450.00 +Notes Payable 44,360.00 +Merchandise Inventory, January 1, 1913 39,783.00 + +Sales 516,875.00 +Purchases 136,375.00 +Prepaid Labor 138,559.00 +Manufacturing Expenses 125,000.00 +Selling Expenses 70,500.00 +Administrative and All Other Expenses 29,799.00 + +$769,625.99 $769,625.99 + +The mortgage is past due—interest 6%—last interest payment, July 1, 1913. + +Accrued interest on notes receivable (good) amounts to $225. +Accrued interest on notes payable amounts to $900. +There were invoices for purchases amounting to $1,800, and wages $2,010, not recorded on books. +Unexpired insurance premiums amount to $350. +Accrued taxes amount to $425. + +There is a chattel mortgage on the machinery securing notes payable, amounting to $16,000, and one of the creditors on open account holds a chattel mortgage of $4,000 on merchandise worth $2,500 given for purchases, the balance due on which is $3,000. + +$7,500 of the good notes receivable have been assigned to secure notes payable for borrowed money. + +72 + +ILLUSTRATIVE PROBLEMS + +The notes receivable are classified: $14,500 good; $1,800 doubtful; and the balance worthless. +The accounts receivable are classified: $40,500 good; $83,500 doubtful; and the balance worthless. + +The land was appraised at $12,000; buildings $25,000; machinery and tools $26,000; furniture and fixtures $600; horses and wagons $1,500; and the merchandise inventory at $32,200. + +The personal estate of Jones consists of a house and lot valued at $18,000, and securities valued at $7,500, and he owes for household debts $750, and to his father-in-law, in notes payable $20,000, for money borrowed from him. + +The personal estate of Smith consists of a house and lot valued at $12,000, upon which there is a mortgage of $5,000; securities valued at $20,000, pledged as collateral for a loan of $15,000. He has other unsecured debts amounting to $2,990. From the foregoing prepare: + +(a) Statement of affairs, Smith and Jones, October 31, 1913, exhibiting thereon also the percentage of their claims likely to be realized by unsecured creditors. + +(b) Deficiency account. (Ohio C.P.A. Examination.) + +Solution to Problem 28 + +ADJUSTING ENTRIES + +October 31, 1913 + + + + + + + + + + + + + + +
Interest on Mortgage Payable (Administrative and Other Expense)$ 500.00
Interest Accrued on Mortgage$ 500.00
Interest accrued on mortgage of $25,050, from July 1, 1913, to October 31, 1913.
+ +**ILLUSTRATIVE PROBLEMS** + +Interest Accrued on Notes Receivable $25.00 +Interest on Notes Receivable $25.00 +Accrued interest on notes receivable. + +Interest on Notes Payable 900.00 +Interest Accrued on Notes Payable 900.00 +Accrued interest on notes payable. + +Purchases 1,800.00 +Accounts Payable 1,800.00 +To enter on the books invoices for purchases which were omitted. + +Productive Labor 8,010.00 +Wages Accrued 8,010.00 +To enter on the books wages accrued and not paid to October 31, 1913. + +Unexpired Insurance 350.00 +Administrative Expense 350.00 +To show unexpired insurance as of October 31, 1913. + +Manufacturing Expenses 445.00 +Taxes Accrued 445.00 +Taxes accrued to October 31, 1913. + +**PROFIT AND LOSS STATEMENT** +January 1, 1913, to October 31, 1913 + +Sales $516,875.00 + +Deduct—Cost of Sales: +Purchases 918,175.00 +Add—Difference in Inventories 27,535.00 +$945,710.00 + +Productive Labor 132,510.00 +Manufacturing Expenses 185,425.00 +$483,635.00 + +Gross Profit on Sales $35,240.00 + +Deduct: +Selling Expenses 70,500.00 +Administrative and All Other Expenses 39,285.00 +$119,785.00 + +Net Loss $67,765.00 + +74 + +**ILLUSTRATIVE PROBLEMS** + +**(a)** + +**SMITH AND JONES** +**STATEMENT OF AFFAIRS** +**October 31, 1913** + + + + + + + + + + + + + + + + + + + + + + + + + + + + + + + + + + + + + + + + + + + + + + + + + + + + + + + + + + + + + + + +






































































































\n\nday\n\n\n\n\n\n\n\n\n\n\n\n\n\n\n\n\n\n\n\n\n\n\n\n\n\n\n\n\n\n\n\n\n\n\n\n\n\n\n\n\n\n\n\n\n\n\n\n\n\n\n\n\n\n\n\n\n\n\n\n\n\n\n\n\n\n\n\n\n\n\n\n +\nday +\nday +\nday +\nday +\nday +\nday +\nday +\nday +\nday +\nday +\nday +\nday +\nday +\nday +\nday +\nday +\nday +\nday +\nday +\nday +\nday +\nday +\nday +\nday +\nday +\nday +\nday +\nday +\dday +\dday +\dday +\dday +\dday +\dday +\dday +\dday +\dday +\dday +\dday +\dday +\dday +\dday +\dday +\dday +\dday +\dday +\dday +\dday +\dday +\dday +\dday +\dday +\dday +\dday +\dday +\dday +\dday +\dday +\dday +\dday +\dday +\dday +\dday +\dday +\dday +\dday +\dday +\dday +\dday +\dday +\dday +\dday +\dday +\dday +\dday +\dday +\dday +\dday +\ddday + +ILLUSTRATIVE PROBLEMS +75 + +| | | +|---|---| +| **35,000.00** | **Machinery, Tools & Appraised Value** | +| Low-Value Collateral: | $65,000.00 | +| Furniture and Fixtures | $85,000.00 | +| $2,500.00 | $1,000.00 | +| Home and Lot | $18,500.00 | +| Securities | $7,500.00 | +| Long-Term Debt: | $18,500.00 | +| Notes Payable | $18,500.00 | +| John Smith - Personal Estate: | $720.00 | +| Home and Lot | $93,720.00 | +| Securities | $4,726.00 | +| Net Cash Equivalents to Secure Loans of $15,000 | $811,666.67 | +| Less Unsecured Personal Debt: | $811,666.67 | + +**$196,666.67** + +**Exempted In-Review** +**Debtors In Review** +**Wages Awarded Claims** +**Taxes Accrued** +**Net Free Assets Applying to Unsecured Creditors, Less 91.69%** +**Deficiency** + +**$8,511,666.67** +**$8,511,666.67** +**$2,533.33** +**$425.00** +**$139,825.25** +**$139,825.25** +**$819,439.43** + +
Book ValueCash$85,000.00Accounts Receivable$65,000.00Good$65,000.00Bad$15,000.00
Accounts Receivable$85,000.00Worshis.$1,800.00Good$83,500.00Bad$1,500.00
Netom Receivable.$18,700.00Loan - Note assigned as security (see contra).$14,900.00Good$14,900.00Bad$4,800.00
Loan - Note assigned as security (see contra).$14,900.00Payable.$7,500.00Good$7,500.00Bad$4,800.00
Demand.$242.66Interest accrued on Note Receivable.$1,862.99Good$1,862.99Bad$242.66
Lien - Chated Mortgage worth $8,555.66.$39,698.99Lien - Chated Mortgage worth $8,555.66.$39,698.99
+ + + + + + + + + + + + + + + + + + + + + + + + + + + + + + + + + + + + + + + + + + + + + + + + + + + + + + + + + + + + + + + + + +
+
DescriptionAmountAmountAmountAmountAmountAmountAmountAmountAmountAmountAmountAmountAmountAmountAmountAmountAmountAmountAmountAmountAmountAmountAmountAmountAmountAmountAmountAmountAmountAmountAmountAmountAmountAmountAmountAmountAmountAmountAmountAmountAmountAmountAmountAmountAmountAmountAmountAmountAmountAmountAmountAmountAmountAmountAmountAmountAmountAmountAmountAmountAmountAmountAmount
Total Debitors In Review:$8,511,666.67
$8,511,666.67
$2,533.33
$425.00
$139,825.25
$139,825.25
$819,439.43
$819,439.43
$819,439.43
$819,439.43
$819,439.43
$819,439.43
$819,439.43
$819,439.43
$819,439.43
$819,439.43
$819,439.43
$819,439.43
$819,439.43
$819,439.43
$819,439.43
$819,439.43
$819,439.43
$819,439.43
$819,439.43
$819,439.43
$819,439.43
$819,439.43
$819,439.43
$819,439.43
$819,439.43
$819,439.43
$819,439.43
$819,439.43
$819,439.43
$819,439.43
$819,439.43
$819,439.43
$819,439.43
$819,439.43
$819,439.43
$819,439.43
$819,439.43
$819,439.43
$819,439.43
$819,439.43
$819,439.43
$819,439.43
$819,439.43
$819,439.43
$819,439.43
$819,439.43
$819,439.43
$819,439.43
$81 + +76 + +ILLUSTRATIVE PROBLEMS + + + + + + + + + + + + + + + + + + + + + + + + + + + + + + + + + + + + + + + + + + + + + + + + + + + + + + + + + + + + + + + + + + + + + + + + + + + + + + + + + + + + + + + + +
Book ValueLiquidityExpected to Rank
$ 83,000.00Mortgage Payable (Deferred income)
506.00Interest Accrued on Mortgage Payable (per contra)
107,500.00Amounts Payable (Deferred income of $4,000 on Merchandise worth) (Deferred per contra)$ 8,500.00
48,500.00Note Payable (Deferred income of $4,000 on Merchandise worth) (Deferred per contra)$ 85,000.00
48,500.00Note Receivable (Deferred income of $4,000 on Merchandise worth) (Deferred per contra)$ 85,000.00
996.00Net Deductions$13,200.00
996.00Accredited Interest on Notes Payable (Deferred per contra)$13,200.00
618.00Preferred Interest (Deferred per contra)$9,924.00
443.00Wages Accrual
Taxes Accrual
Total$136,459.00
Capital Obligations:
Funds - Bank, Government$35,596.00
Funds - Trade Receivables$26,992.08
Total A-1 Statement$62,588.08
Date - Current period$131,175.56

$199,636.59
$199,636.59 + +ILLUSTRATIVE PROBLEMS 77 + +(b) + + + + + + + + + + + + + + + + + + + + + + + + + + + + + + + + + + + + + + + + + + + + + + + + + + + + + + + + + + + + + + + + + + + + + + + + + + + + + + + + + + +
DEFICIENCYACCOUNT
Loss on Accounts ReceivableAppreciation of Land$ 8,000.00
8,140.00William Jones
Loss on Notes ReceivableInvestment45,000.00
ablePersonal Estate4,756.00
4,400.00John Doe
Loss on Unexpired Insurance350.00Investment35,000.00
Personal Estate9,918.00
Loss on Real Estate5,300.00
Deficiency11,225.00
Total Losses9,900.00
Loss on Furniture and Fixtures990.00
Loss on Tractors and Wagons850.00
Net Trading Loss for period67,763.00
+
$196,985.00$196,985.00

+
+ +Points Illustrated in Problem 28 + +(a) Rights of Personal Creditors. It will be noted that the personal estate of each partner is listed to produce the value of his assets minus his personal liabilities. This follows from the fact that under the procedure of filing individual petitions in bankruptcy the personal creditors would be given precedence over the firm creditors so far as the personal estate was available (see Volume I, Chapter XXII, § 4, and Chapter XXXIII, § 3). + +(b) Interest on Mortgage. The security for the principal of the mortgage covers also the interest. See comments on Problem 87. + +(c) Appreciation of Land. See comments on Problem 27 for a similar situation. + +(d) Unexpired Insurance. In this case the unexpired insurance premiums are stated as having no realizable value. See the discussion of this point under Problem 27. + +78 +ILLUSTRATIVE PROBLEMS + +(e) Valuation of Accounts Receivable. The doubtful accounts receivable are stated as having no realizable value because the problem does not give any indication of the percentage which might be recovered. In a case of this sort it is better to be conservative in the statement of realizable value because otherwise the receiver or trustee would be overcharged and the loss on realization of assets would be overstated. + +(f) Merchandise Inventory. The problem states that the merchandise inventory was appraised at $83,200. This amount is stated as realizable because it is assumed that the appraisal was on the liquidating basis and not on the assumption that the business would continue. Consequently, a loss on realization of merchandise is shown. It is more customary to value the merchandise on a going concern basis and then to appraise it on the liquidating basis in order to indicate the loss on realization. Since the problem does not give this information the solution cannot supply it. + +Problem 29 + +The affairs of Peter Post, a manufacturer, were in a very critical condition, for, although he had an unimpaired investment of $62,500, and his books showed a clear increase of $6,022, he owed his trade creditors $25,289 and had only $245 in cash and $4,062 in receivable book accounts on which to rely for funds. The rest of his business estate was tied up in the following chattels which he had acquired in an effort to keep pace with a business growth that had outrun his capital: machinery and tools $31,497; raw materials $18,838; partly made goods $31,502; and finished wares $7,587. + +ILLUSTRATIVE PROBLEMS 79 + +It was also necessary, in order to continue operations, to have immediate cash for pay-rolls and incidental expenses. + +A meeting of his principal creditors was called, and, as it appeared that the business was well established, profitable, and had a sure and growing market, they decided to advance him $6,000 in cash for immediate needs and extend his credit in a sufficient amount to permit of the purchase of necessary materials and generally to continue operations till the present stock of materials could be made up and realized on. + +In order to insure the proper application of the funds and credit so provided, a trustee was appointed to administer the finances till the creditors' claims were satisfied, at which time the control would revert to the proprietor. + +The subsequent operations under the trusteeship were as follows: cash paid for labor $15,725; for expenses $5,490; for additional tools $8750; purchases on book account, charged to materials $6,300, to expenses $15,000; sales on book account $72,300; loss on collection on book debts $890; personal drawings of Peter Post $3,500. + +The unliquidated values at the close of the trusteeship were as follows: inventory of raw materials $5,000; finished wares $27,900; accounts receivable outstanding $3,382; and accounts payable 889. + +Prepare, with due regard to the grouping, order, and arrangement of the items, as best calculated clearly to display the facts, (a) realization and liquidation account, (b) trustee's cash account, (c) balance sheet of business as restored to Peter Post. + +(New York C. P. A. Examination.) + +80 + +**ILLUSTRATIVE PROBLEMS** + +**Solution to Problem 29** + +**PETER POST** + +**REALIZATION AND LIQUIDATION ACCOUNT** + + + + + + + + + + + + + + + + + + + + + + + + + + + + + + + + + + + + + + + + + + + + + + + + + + + + + + + + + + + + + + + + + + + + + + + + + +
















































+ + + + + +
Assets to be Realized:Liabilities to be Liquidated:
Machinery and Tools$ 31,497.00Trade Credits$ 35,858.00
Raw Material6,360.00Assets Realized:
Goods in Process31,362.00By Trustee's Trading Account:
Finished Goods7,207.00Trade Credits$15,858.00
Accounts Receivable4,842.00Goods in Process$1,518.00
Total$ 35,962.00Finished Goods7,207.00
Liquidity Liquidated:Total Assets Realized:$15,858.00
Trade Credits$13,919.00Accounts Receivable:$9,992.00 $1,668.00
Assets Not Realized:Machinery and Tools:31,497.00
Loss on Realization, per Balance Sheet:$98.00
$118,835.00$118,835.00
TRUSTEE'S TRADING ACCOUNT
Sales
+ + + + + + + + + + + + + + + + + + + + + + + + + + + + + + + + + + + + + + + + + + + + + + + + + + + + + + + + + + + + + + + + + + + + + + + + + + + +
DescriptionTotal Amount
+ +Taxes in Excess of Estimated Amount +39.0 + +Liabilities to be Liquidated: +Mortgage Payable +$196,666.67 +Notes Payable +133,555.56 +Accounts Payable +132,444.44 +Interest Accrued on Mortgage +142,222.22 +Taxes Accrued (estimated) +835.83 +$777,113.83 + + + + + + + + + + + + + + + + + + + + + + + + + + + + + + + + + + + + + + + + + + + + + + +
86ILLUSTRATIVE PROBLEMS
Assets Realized:$ 15,000.00
Notes Receivable
Accounts Receivable105,500.00
Materials and Supplies, per Trustee's Trading Account:95,000.00
Assets Not Realized:$16,300.00
Land and Buildings8125,000.00
Machinery and Tools75,000.00
Furniture and Fixtures10,000.00
Loss on Realization and Liquidation, per Balance Sheet:8,330.00
$777,115.00
+ +X, Y, AND Z + +TRUSTEE'S TRADING ACCOUNT + +For the Year Ended December 31, 1908 + + + + + + + + + + + + + + + + + + + + + + + + + + + + + + + +
+ + +$45,000.00                                                                               
+ + +$5,000.00               
+ + +$5,806.67  
+ + +$2,893.33&nb
+ + +$62.67&nb
+ + +$366.67&nb
+ + +$48,475.67&nb
+ + +
Sales:8340,435.00
Cost of Goods Sold:
Inventory, January 1, 1908, per Realization and Liquidation Statement:95,000.00
Purchases:88,000.00
Labor:153,000.00
Total Inventory, December 31, 1988, per Balance Sheet:$328,000.00
Gross Profit:$356,000.00
Expenses:
General Expense:
Interest on Mortgage:
Interest on Notes Payable:
Bad Debt Allowance Off:
Taxes (estimated):
Discounts and Allowances:
Net Profit, per Balance Sheet:

+ +ILLUSTRATIVE PROBLEMS +X, Y, AND Z + +BALANCE SHEET +December 31, 1908 + + + + + + + + + + + + + + + + + + + + + + + + + + + + + + + + + + + + + + + + + + + + + + + + + + + + + + + + + + + + + + + + + + + + + + + + + + + + + + + + + + + + + + + + + + + + + + + + + + + + + +
Assets
Fixed Assets:
Land and Buildings$185,000.00
Machinery and Tools75,000.00
Furniture and Fixture10,000.00$180,000.00
Current Assets:
Cash6,220.00
Accounts Receivable100,790.00
Notes Receivable6,590.00
Merchandise96,000.0095,420.00
Total Assets$413,429.00
Liabilities and Capital
Fixed Liabilities:
Mortgage Payable$180,000.00
Current Liabilities:
Notes Payable$185,000.00
Interest Accrued on Mortgage1,250.00$85,115.00$85,115.00 +
+ + + +
Balance, January 1, 1908
+ + + +
Profit on Trading
+ + + +
Loss on Realization and Liquidation, per Realization and Liquidation Account
+ + + +
Balance, December 31, 1908
+ + + +
Total Liabilities and Capital
+ + + +
$85,115.00
+ + + +
$85,115.00
+ + + +
$85,115.00
+ + + +
$85,115.00
+ + + +
$85,115.00
+ + + +
$85,115.00
+ + + +
$85,115.00
+ + + +
$85,115.00
+ + + +
$85,115.00
+ + + +
$85,115.00
+ + + +
$85,115.00
+ + + +
$85,115.00
+ + + +
$85,115.00
+ + + +
$85,115.00
+ + + +
$85,115.00
+ + + +
$85,115.00
+ + + +
$85,115.00
+ + + +
$85,115.00
+ + + +
$85,115.00
+ + + +
$85,115.00
+ + + +
$85,115.00
+ + + +
$85,115. + +88 +ILLUSTRATIVE PROBLEMS + +Points Illustrated in Problem 30 + +(a) Realization and Liquidation Statement. The reasons for the technical form of this statement are explained in Volume I, Chapter XIII, § 3. + +(b) Trading Result. The necessity for distinguishing the result of trading from the loss or gain on realization is pointed out in Volume IV, Chapter XXV, § 2. + +(c) Form of the Realization and Liquidation statement given in the solution is a simple form following the lines laid down in Volume I, Chapter IX, § 15. + +(d) Accrued Liability for Taxes. The desirability of setting up an estimated amount for this liability is pointed out in Volume IV, Chapter III, § 1. + +(e) Supplementary Charges. The purpose of the section of the realization and liquidation account entitled "Supplementary Charges" is explained in Volume I, Chapter XIII, § 12. + +(f) Omission of Cash. See the discussion on this point under Problem 29. + +--- + +Problem 31 + +Smith and Murray have been doing business as equal partners and have kept their books by single entry. They wish to admit Davis as a partner and have their books kept by double entry. Their books and inventory taken show the following assets and liabilities: merchandise $9,241; cash $850; real estate $3,000; accounts receivable $6,941; store fixtures $571; Smith's investment account, credit $6,400; Murray's investment account, credit $5,390; accounts payable $4,175; notes payable $875. + +ILLUSTRATIVE PROBLEMS 89 + +Prepare statement of assets and liabilities and find each partner's present worth, after which make opening entry for the double-entry set of books. Davis is admitted and invests cash $3,000; merchandise $2,000; notes receivable $1,000. Make opening entry for Davis. + +**Solution to Problem 31** + +**SMITH AND MURRAY** +**STATEMENT OF ASSETS AND LIABILITIES** + + + + + + + + + + + + + + + + + + + + + + + + + + + + + + + + + + + + + + + + + + + + + + + + + + +
Assets
Cash ..$ 850.00
Accounts Receivable6,941.00
Merchandise14,414.00
Real Estate ..0,000.00
Store Fixtures571.00
Total Assets$80,683.00
Liabilities
Notes Payable$ 975.00
Accounts Payable4,175.00
Total Liabilities5,150.00
Net Worth815,453.00
+ +**STATEMENT SHOWING NET PROFIT** + + + + + + + + + + + + + + + + + + + + + + + + + + + + + + + + + + +
Net Worth$15,453.00
Deduct—Net Balances of Capital Accounts:
Smith ..$6,400.00
Murray ..5,390.00
Net Profit:
Smith, one-half ..$1,831.50
Murray, one-half ..1,831.50
$3,663.00
+ +90 + +# ILLUSTRATIVE PROBLEMS + +## OPENING ENTRIES + + + + + + + + + + + + + + + + + + + + + + + + + + + + + + + + + + + + + + + + + + + + + + + + + + + + + + +
Cash$ 856.00
Accounts Receivable6,041.00
Merchandise8,341.00
Store Fixtures571.00
Real Estate3,000.00
Notes Payable$ 975.00
Accounts Payable4,175.00
Smith, Capital8,231.50
Murray, Capital7,281.50
Cash3,000.00
Notes Receivable1,000.00
Merchandise4,000.00
Davis, Capital6,000.00
+ +**Points Illustrated in Problem 31** + +(a) Single-Entry Statement. The preparation of statements as required in this problem, where the books of account have not been kept by double entry, is outlined in Volume I, Chapter XXXV, § 9. + +(b) Opening Entry for Partnership. The entries required for opening partnership accounts are outlined in Volume I, Chapter XXXL § 3. + +--- + +**Problem 32** + +On January 1, 1920, Robert A. Grant began business as a retail dry goods merchant. His capital at the time consisted of the following assets: merchandise $12,300, cash $1,150, furniture and fixtures $600. There were no liabilities to be recorded at the outset of the business enterprise. + +He sold most of his goods for cash, although credit was extended in certain cases. + +ILLUSTRATIVE PROBLEMS 91 + +The books of account, which had been kept by single entry, consisted of a ledger and a cash book, and various supporting memorandum records, which, however, from their nature were not entirely reliable. + +At the end of three months, Mr. Grant desired to ascertain whether he was making any money. The clerks were set to work taking inventory, and the book-keeper was instructed to prepare a list showing all outstanding accounts receivable and payable, and all other assets not recorded in the accounts. This produced the following results: + + + + + + + + + + + + + + + + + + + + + + +
Merchandise on Hand$24,002.62
Accounts Receivable4,163.74
Accounts Payable15,893.21
Cash in Bank4,374.48
Cash in Drawer241.17
+ +Purchases of office equipment during the period amounting to $827.5 were disclosed by an inspection of the paid invoices on file. + +Invoices have been received and entered on the books covering the purchase of goods amounting to $875.20. The goods had not yet arrived and accordingly were not included in the inventory. + +Feeling the need of more working capital, Mr. Grant sold, on February 10, certain bonds which he had been holding as investments, realizing thereon $1,250, which amount was placed in the business. + +Prepare: +(a) Statements showing the assets and liabilities and the net profit or loss for the period. +(b) Entry to open double-entry books. + +92 + +**ILLUSTRATIVE PROBLEMS** + +**Solution to Problem 32** + +**(a)** + +ROBERT A. GRANT + +**COMPARATIVE STATEMENT OF ASSETS AND LIABILITIES** + +For Three Months + + + + + + + + + + + + + + + + + + + + + + + + + + + + + + + + + + + + + + + + + + + + + + + + +
AssetsJan. 1Mar. 31Increase
Cash in Bank$ 1,150.00$ 2,574.43$ 1,424.43
Inventory18,300.0026,437.878,137.87
Accounts Receivable$ 2,165.74$ 2,165.74
Cash in Drawer$ 244.17$ 244.17
Furniture and Fixtures600.00875.00$ 275.00
Total Assets$14,050.00$30,275.16$16,225.16
+ + + + + + + + + + + + + + + + + + + + + + + + + + + + + + + + + + + + + + + + + +
Liabilities
Accounts Payable$15,203.21$15,203.21
Total Liabilities$15,203.21
Net Worth$14,050.00$15,071.95
Net Increase in Net Worth$ 1,021.95
Deduct-Additional Investment-1,250.00
Net Loss$ 285.05
+ +**(b)** + +March 31, 1989 + + +
Cash in Bank8 $ 8,574.43
Cash in Drawer$ 244.17
Inventory$ 4,437.88
Accounts Receivable$ 2,165.74
Furniture and Fixtures875.00
Robert A. Grant, Capital$15,071.95
Accounts Payable15,203.21
+ +To change from single to double entry. + +A table showing comparative financial statements. + +ILLUSTRATIVE PROBLEMS + +93 + +**Points Illustrated in Problem 32** + +(a) Single-Entry Statements. The method of preparing statements of profit and loss from single-entry accounts is described in Volume I, Chapter XXXV, § 7. This method would necessarily be used wherever the accounts had not been kept by double entry. + +(b) Invoices for Merchandise Not Received. It is necessary at the close of each fiscal period to ascertain that all invoices for merchandise purchased but not received are correctly recorded in the books. In view of the fact that the invoice represents a current account payable, whereas the merchandise, although a current asset, must be sold before cash can be realized upon it, it is necessary to record both the merchandise in the inventory and the invoice among the accounts payable. Otherwise, the true financial condition of the business cannot be indicated. + +(d) Furniture and Fixtures. The student is invited to the fact that the purchase of furniture and fixtures in this example does not represent an expense of the business. (See Volume I, Chapter IX, § 5, and Volume IV, Chapter IV, § 13.) + +(e) Capital Contributions. This problem illustrates the fact that the contribution by a proprietor of additional capital does not constitute income of the business. This is explained in Volume I, Chapter III, § 4, and Chapter X, § 13. + +--- + +**Problem 33** + +A “single-entry” set of books for 1912 are sent to you with an order to prepare a profit and loss statement for the year and a balance sheet at December 31. The starting capital was $834,500. + +94 + +**ILLUSTRATIVE PROBLEMS** + + + + + + + + + + + + + + + + + + + + + + + + + + + + + + + + + + + + + +
The accounts receivableJan. 1$65,500.00Dec. 31$44,000.00
The accounts payableJan. 17,500.00Dec. 316,750.00
The merchandiseJan. 18,500.00Dec. 319,500.00
The plant and machineryJan. 110,000.00Dec. 3110,000.00
The furniture and fixturesJan. 1755.00Dec. 31755.00
+ +A summary of cash book for the year shows as follows: + + + + + + + + + + + + + + + + + +
Received:
Accounts Receivable$83,000.00
Capital Paid In$2,500.00
Disbursed:
Bank Overdraft, January 1$3,700.00
Accounts Payable$2,655.00
General Reserve Fund5,995.00
Wages7,725.00
Personal Account1,500.00
Leaving a bank account of $8,495, and currency on hand $85.
+ +Provide 3% interest on capital, disregarding additions during the year and personal drafts, deducting 10% for plant and machinery depreciation, 5% for furniture and fixtures, and 5% for bad debts reserve. + +(Illinois C. P. A. Examination.) + +**Solution to Problem 33** + +**COMPARATIVE STATEMENT OF ASSETS AND LIABILITIES** + + + + + + + + + + + + + + + + + + + + + + + + + + + + + + + + + +
AssetsJan. 1Dec. 31Increase Decrease
Cash$2,525.00$2,625.00$125.00
Accounts Receivable$26,500.00$44,125.00$17,625.00
Merchandise$8,525.00$9,995.00$1,475.00
Plant and Machinery$19,995.00$21,875.00
+
Total Assets (Dec.)$84,995)$86,875)$178)
+ +Total Assets +$84,995 +$86,875 +$178 + +ILLUSTRATIVE PROBLEMS 95 + + + + + + + + + + + + + + + + + + + + + + + + + + + + + + + + + + + + + + + + + + + + + + + + + + + + + + + + + + + + + + + + + + + + + +
LiabilitiesJan. 1Dec. 31IncreaseDecrease
Accounts Payable$ 7,500.00$ 9,750.00$ 2,250.00
Bank Overdraft3,700.003,700.00
Reserve for Bad Debts2,800.002,800.00
Total Liabilities$11,200.00$11,550.00$750.00
Net Worth834,500.00837,265.00818,765.00
Net Increase in Net Worth$18,765.00
Add—Drawings1,365.00
Less:$20,465.00
Capital Paid In$2,500.00
+
DescriptionAmount
Merchandise on Hand, January 18,500.00
Net Purchases14,735.00
Less—Merchandise on Hand, December 31-821,455.00
Gross Profit on Sales$35,735.00
                                                             
 
 
 
 
                  &nb + +96 + +# ILLUSTRATIVE PROBLEMS + +## BALANCE SHEET +December 31 + +**Assets** + +| Cash | $ 2,050.00 | +|---|---| +| Accounts Receivable | $44,000.00 | +| Less—Reserve for Bad Debts | 2,800.00 | $41,800.00 | +| Merchandise on Hand | 9,500.00 | +| Plant and Machinery | 9,000.00 | +| Furniture and Fixtures | 655.00 | +| **Total Assets** | **$63,015.00** | + +**Liabilities and Net Worth** + +| Liabilities—Accounts Payable | $ 9,750.00 | +|---|---| +| Net Worth: Capital, January 1 | $84,500.00 | +| Add: Net Profit for the year | $16,500.00 | +| Capital Paid In | 8,500.00 | +| 5% on Capital of January 1 | 1,755.00 | $9,265.00 | +| Less—Drawings | 1,500.00 | $73,255.00 | +| **Total Liabilities and Net Worth** | **$83,015.00** | + +## Points Illustrated in Problem 33 + +(a) Single-Entry Statements. For the method of preparing statements from single-entry records, see Volume I, Chapter XXXV, § 7. + +(b) Proprietor's Drawings. The fact that drawings by the proprietor are not expenses of the business and must not be so construed in preparing single-entry statements is discussed in Volume I, Chapter XXXV, § 7. + +(c) Interest on Capital. This problem illustrates the fact that calculation of interest on capital has no effect upon the + +ILLUSTRATIVE PROBLEMS + +97 + +net profit of the business. This is further described in Volume I, Chapter XXXI, § 10. + +(d) Depreciation and Bad Debts. The principles underlying the handling of these items are described in Volume I, Chapter XXXIX, and in Volume IV, Chapter XII. + +(e) Depreciation of Plant and Machinery. In this problem the plant and machinery and the furniture and fixtures are stated in the balance sheet at their net amounts after the deduction of the amount of depreciation. This is generally not considered good practice. Instead, reserve accounts should be set up to record the depreciation. This is explained in Volume I, Chapter XXXIX, and in Volume IV, Chapter XII. + +(f) Calculation of Net Sales. In this problem the difference between the amount of net sales is determined by adding the cash collections to the increase in the accounts receivable. The calculation of net purchases is similarly made by adding the increase in accounts payable and the cash payments. This latter step is based on the assumption that all the accounts payable are incurred for merchandise purchases. + +--- + +**Problem 34** + +You are asked to prepare an account showing the profit earned by a concern for a certain period. The books have been kept by single entry and you gather from them the following: + + + + + + + + + + + + + + + + + + + + + + + + + + + + + + + + + + +
Capital818,259.00
Cash6,500.00
Accounts Receivable13,500.00
Merchandise14,490.00
Furniture6,500.00
Accounts Payable3,530.00
Bills Payable3,000.00
Merchandise used by Proprietor860.00
+ +2 + +98 + +**ILLUSTRATIVE PROBLEMS** + +The capital above set out is the balance of the account after $1,500 withdrawn during the period, and $1,200 for salary, have been charged up against it. + +Set up the Profit and Loss account. + +(NY York C. P. A. Examination.) + +**Solution to Problem 34** + +**CALCULATION OF NET PROFIT** + + + + + + + + + + + + + + + + + + + + + + + + + + + + + + + + + + + + + + + + + + + + + + + + + + +
Capital at End of Period:
Assets:
Cash$ 600.00
Accounts Receivable15,600.00
Merchandise10,400.00
Fixtures1,630.00


Liabilities:
Accounts Payable$ 830.00
Bills Payable5,000.00


Capital$21,400.00
+ + + + + + + + + + + + + + + + + + + + + + + + + + + + + + + + + + + + + + + + + + + + + + + +
Capital at Beginning of Period:
Ledger Balance$18,960.00
Add:
Cash Drawings by Proprietor$1,500.00
Salary Drawn by Proprietor1,200.00


Decrease in Capital$660.00
Deduct:
Cash Drawings by Proprietor$1,500.00
Salary Drawn by Proprietor1,200.00
Merchandise Drawn by Proprietor800.00
Net Profit for the Period$2,540.00
+ +**Points Illustrated in Problem 34** + +(a) Profit and Loss Account. In this problem the State Board of Examiners ask for a Profit and Loss account when obviously what is meant is, not an account, but a statement showing + +ILLUSTRATIVE PROBLEMS 99 + +the calculation of profit. As pointed out in Volume I, Chapter IX, § 6, care must be exercised when a Profit and Loss account is asked for to determine whether or not an account is really required. In this case, as explained in Volume I, Chapter XXXV, § 11, it would be impossible to set up a Profit and Loss account, strictly speaking, because no nominal accounts have been kept and the factors or elements that went into a Profit and Loss account are not known. Accordingly, the solution does not show an account but a calculation. + +(b) Inadequacy of Single Entry. This problem illustrates the inadequacy of profit and loss statements prepared from single-entry accounts. While the net profit can be ascertained, the various elements that cause it and the various kinds of expenses that affect it cannot be known. This inadequacy is one of the reasons why double-entry book-keeping is almost universally preferred. (See Volume I, Chapter XXXV, § 11.) + +(c) Capital. Capital is the excess of assets over liabilities, as explained in Volume I, Chapter III, § 1. + +(d) Bills Payable. As explained in Volume I, Chapter XXVII, §§ 2 and 3, it is better to use the term "notes payable" instead of bills payable. + +(e) Proprietor's Salary. The fact that this so-called salary is actually a drawing on account of profits and does not constitute a real expense of the business is explained in Volume I, Chapter XXXI, § 12. It is customary in many businesses to record these so-called salaries for statistical purposes and to enable one partner to draw more than another when he renders greater or more valuable service. + +(f) Merchandise Inventory. In order to ascertain whether the $800 is the cost or the selling price of the merchandise. When a proprietor withdraws merchandise for his own use he should be charged with it at cost, care being taken not to include such drawings among the regular sales of the business. This matter is explained in Volume I, Chapter X, § 20. + +100 +ILLUSTRATIVE PROBLEMS + +(g) Decrease in Capital. Notwithstanding the fact that there was a net profit of $2,840, the capital in the business decreased $660. This resulted from the fact that the cash drawings and salary of the proprietor together with the merchandise withdrawn by him exceeded the net profits of the business. The problem illustrates the fact that the decrease or increase in capital does not indicate the net profit or loss. (See Volume I, Chapter XXXV.) + +**Problem 35** + +Classify and group the following accounts of a manufacturing company according to kind of asset, liability, loss, and gain: + +1. Accounts Payable +2. Accounts Receivable +3. Accrued Salaries and Wages +4. Advertising +5. Bad Debts Written Off +6. Bills Payable +7. Bills Receivable +8. Bond Discount +9. Bond Premium +10. Bond Interest Accrued +11. Capital Stock +12. Cash +13. Credit Department Expenses +14. Depreciation of Buildings, Ma- +chinery, and Plant +15. Depreciation of Workmen's Cot- +tage +16. Directors' Fees +17. Discount on Purchases +18. Discount on Sales +19. Federal Income Tax +20. First Mortgage Bonds +21. Freight and Cartage Inward +22. Freight and Cartage Outward +23. General Office Expenses +24. Good-Will +25. Insurance +26. Insurance Premiums Unexpired +27. Interest on Bills Payable +28. Interest on Bonds +29. Investments in Investments +30. Inventory, Raw Materials +31. Inventory, Goods in Process +32. Inventory, Manufactured Goods +33. Inventory, Finished Goods +34. Maintenance of Buildings, Ma- +chinery, and Plant +35. Maintenance of Workmen's Cot- +tage +36. Manufacturing Power, Heat, and Light +37. Miscellaneous Factory Expenses +38. Miscellaneous Selling Expenses +39. Non-productive Labor Other Equipment +40. Office Supplies +41. Office Equipment +42. Officer Salaries and Expenses +43. Organization Expenses + +ILLUSTRATIVE PROBLEMS 101 + + + + + + + + + + + + + + + + + + + + + + + + + + + + + + + + + + + + + + + + + + + + + + + + + + + + + + + + + + +
44. Patent Rights53. Reserve for Doubtful Accounts
45. Patterns and Drawings56. Reserve for Sinking Fund
46. Plant Site57. Returns and Allowances on Purchases
47. Plant Buildings58. Returns and Allowances on Sales
48. Plant, Machinery, and Equipment59. Sales of Waste Material
49. Workmen's Cottages60. Sales of Waste Material
50. Purchasing Department Expenses61. Sales Agents Commissions
51. Raw Materials Purchased62. Salaries and Wages
52. Rent of Workmen's Cottages63. Salaries and Wages Expenses
53. Reserve for Depreciation of Buildings, Machinery, and Plant64. Sinking Fund Investments
54. Reserve for Depreciation of Workmen's Cottages65. Surplus
66. Surplus from Plant and Equipment
67. Taxes Accrued
68. Workmen's Cottages (Wisconsin C.P.A. Examination.)
+ +**Solution to Problem 35** + +**Assets** + +Fixed Assets: +- **46. Plant Site** +- **47. Plant Buildings** +- **48. Plant, Machinery, and Equipment** +- **49. Workmen's Cottages** +- **40. Office Equipment** +- **41. Goods in Transit** +- **44. Patent Rights** +- **45. Patterns and Drawings** + +Investments: +- **33. Investments (Outside)** +- **64. Sinking Fund Investments** + +Current Assets: +- **12. Cash** +- **2. Accounts Receivable** +- **7. Bills Receivable** + +102 +ILLUSTRATIVE PROBLEMS + +Current Assets—Continued +Inventory: +30. Inventory, Raw Materials +31. Inventory, Goods in Process +32. Inventory, Manufactured Goods + +Deferred Charges and Prepaid Items: +45. Organization Expenses +46. Insurance Premiums Unexpired + +Liabilities and Capital +Fixed Liabilities: +30. First Mortgage Bonds + +Current Liabilities: +1. Accounts Payable +6. Bills Payable + +Accrued Items: +3. Accrued Salaries and Wages +10. Bond Interest Accrued +67. Taxes Accrued + +Reserves: +53. Reserve for Depreciation of Buildings, Machinery, and Plant +54. Reserve for Depreciation of Workmen's Cottages +55. Reserve for Doubtful Accounts +56. Reserve for Sinking Fund + +Capital: +11. Capital Stock +65. Surplus + +Income and Expense +Manufacturing Cost Accounts: +49. Productive Labor +51. Raw Materials Purchased (after elimination of inventory) + +ILLUSTRATIVE PROBLEMS 103 + +Manufacturing Cost Accounts—Continued + +21. Freight and Cartage Inward +57. Returns and Allowances on Purchases +58. Purchasing Department Expenses + +Manufacturing Expenses: + +39. Non-productive Labor +36. Maintenance of Buildings, Machinery, and Plant +36. Manufacturing Power, Heat, and Light +66. Taxes on Plant and Equipment +25. Insurance +14. Depreciation of Buildings, Machinery, and Plant +37. Miscellaneous Factory Expenses +15. Depreciation of Workmen's Cottages +35. Maintenance of Workmen's Cottages + +Deductions from Manufacturing Cost: +52. Rent of Workmen's Cottages +60. Sales of Waste Material + +Trading Accounts: +59. Sales of Manufactured Goods +58. Returns and Allowances on Sales + +Selling Expenses: +4. Advertising +62. Salesmen's Salaries +61. Sales Agents' Commissions +63. Salesmen's Expenses +13. Credit Department Expenses +22. Freight and Cartage Outward +58. Miscellaneous Selling Expenses + +General Administrative Expenses: +44. Officers' Salaries and Expenses +16. Directors' Fees +41. Office Salaries +25. General Office Expenses +19. Federal Corporation Tax + +104 + +# ILLUSTRATIVE PROBLEMS + +**Capital Expenses:** +18. Discount on Sales +5. Bad Debts Written Off +27. Interest on Bills Payable +28. Bond Discount +28. Interest on Bonds + +**Capital Income:** +17. Discount on Purchases +29. Income from Investments +9. Bond Premium + +**Points Illustrated in Problem 35** + +(a) General Classification of Accounts. This problem illustrates the general classification of accounts as described in Volume I, Chapter VI. + +(b) Working Capital. The solution is arranged to indicate the excess of current assets over current liabilities, which constitutes the working capital. The necessity for this form is explained in Volume I, Chapter XI, § 14. + +(c) Separate Account for Buildings. The buildings are represented by a separate account, but the desirability of this is pointed out in Volume IV, Chapter XVII, § 4. + +(d) Bills Receivable and Payable. Attention is invited to the fact that this problem refers to "bills receivable" and "bills payable." It is generally better to designate such items by the terms "notes receivable" and "notes payable," as pointed out in Volume I, Chapter XXVII, § 3. +As this problem was set by a state board of examiners, the terminology used was correct. It would be advisable, however, to use the term "notes" instead of "bills." Historical explanation of the latter term is given in the chapter to which reference is here made. + +(e) Deferred Charges. The advisability of setting up deferred charges is pointed out in Volume IV, Chapter II, § 3. + +(f) Accrued Liabilities. The desirability of recording accrued liabilities is pointed out in Volume IV, Chapter III, § 1. + +ILLUSTRATIVE PROBLEMS + +(g) Separate Reserve Accounts. This problem illustrates the use of separate reserves as advocated in Volume I, Chapter XXIX, § 4. + +(h) Balance Sheet Treatment of Reserves. In this solution the reserves for depreciation and for doubtful accounts are shown on the balance sheet under the capital stock, but it is generally considered better to deduct them from the asset accounts to which they apply. This matter is discussed in Volume IV, Chapter VIII, § 2. The reserve for sinking fund may properly be shown in the position in which it appears in this solution. This kind of reserve is discussed in Volume IV, Chapter XI, § 10. + +(i) Capital Stock and Surplus. It must be noted that the capital stock and the surplus are combined to show the total capital. This is in accordance with the principles stated in Volume I, Chapter II, § 8. + +(j) Separate Account for Surplus. The necessity for recording the surplus in a separate account is explained in Volume I, Chapter XXXIII, § 3. + +(k) Freight and Cartage Inward. This item is treated as a part of the cost of goods sold in accordance with the principles explained in Volume I, Chapter IX, § 8. + +(l) Depreciation. The depreciation of buildings, machinery, and plant is treated as a manufacturing expense in accordance with Volume I, Chapter XXIX, §§ 1 and 4. + +(m) Freight and Cartage Outward. This item is shown as a selling expense but by some accountants it is usually treated as a deduction from sales price. Because of the problem this is discussed in Vol. I, Chapter IX, § 7. + +(n) Cash Discounts. Cash discounts on sales and purchases are shown as capital expenses and capital income respectively. This is considered better than to show them as adjustments of the sale or purchase price. (See Volume I, Chapter XXV, § 8.) + +(o) Bond Discount and Premium. These items appear under capital expense and capital income respectively. As pointed out in Volume IV, Chapter X, § 10, they are pro- + +106 +ILLUSTRATIVE PROBLEMS + +ratable adjustments of the bond interest and should be extended over the life of the bonds. +(p) Income from Investments. This item is properly shown as capital income because it is extraneous to the operations of the business. This matter is discussed in Volume I, Chapter IX, § 12. + +(q) Workmen's Cottages. The accounts concerning workmen's cottages may be treated either as operating or as extraneous matter. In a specific case the decision should be made in consideration of the purpose of the cottages. If they satisfy all the test of workmen's cottages, good living conditions which are vital to the quality of the daily labor, the operation of the cottages may be considered a manufacturing or operating item. In any event, the rent income is a reduction of the expense involved and if the latter is regarded as a manufacturing or operating item the rent income should not be considered as capital income. +The net result is that this item is shown as a manufacturing expense. + +(r) Taxes and Insurance on Plant and Equipment. These two items are shown as manufacturing expenses. Economists would be likely to regard them as capital expenses. A discussion of the reasons for such treatment can be found in any standard text-book on economics, but as it is beyond the scope of this set this point is not elaborated in any of these volumes. From the point of view of the business enterprise these items may properly be regarded as manufacturing expenses. + +(s) Federal Corporation Tax. This item is shown as a general administrative expense. The same comments apply as are noted in the preceding point. + +--- + +**Problem 36** + +Goods are supplied to a branch at 120% of cost, or $824,000. The branch reports an inventory of $15,000; + +ILLUSTRATIVE PROBLEMS 107 + +sales $30,000; expenses paid $8,000; collections $10,500; and bad debts $750. + +Formulate all entries necessary to record transactions, including entries to close accounts of branch for year to show profit on branch home office books. + +**Solution to Problem 36** + +Branch A. + + + + + + + + + + + + + +
Purchase$24,000.00
Branch A. Contingent Profit$28,000.00
To record transfer of merchandise of $20,000 at 20% above coat.$4,000.00
+ +Branch A. + + + + + + + + + + + + + + + + + + + + + + + + + + + + + + + + + +
Branch A. Profit and Loss$6,250.00
For Sales$30,000.00
Less Inventory$15,000.00
Gross Profit$15,000.00
Less:
Expense$8,500.00
Bad Debts750.00
Total Contingent Profit$8,650.00
+ +To record operating result reported by Branch A. + +Branch A. Contingent Profit +Branch A. Profit and Loss + +$1,500.00 +$1,508.99 + +For transfer the portion of contingent profit applicable to sales made by Branch A. + +**Points Illustrated in Problem 36** + +(a) Controlling Account. The account with Branch A, which is kept on the books of the home office, is a controlling account which controls the branch ledger. A trial balance of the latter book should indicate a net debit or credit equal to the balance of the controlling account. (See Volume I, Chapter XXIV, § 13.) + +(b) Deferred Credit. This is an illustration of the necessity for + +108 + +ILLUSTRATIVE PROBLEMS + +deferring the credit in the case of an income which is not actually earned during the period. The excess of the amount at which goods are billed to Branch A over the cost of the goods is credited to a contingent profit account instead of to Sales or a present profit. (See Volume IV, Chapter IV, § 6.) + +--- + +**Problem 37** + +A firm having several branches maintains an account with each branch in the ledger and charges to such account all goods sent to the agents for stock. When stock is taken the balance of each branch account is treated as ordinary accounts receivable, and is included in the general debts owing to the firm. + +What objections are there to this method? If any, state how you would deal with the accounts. + +(Ohio C. P. A. Examination.) + +**Solution to Problem 37** + +The objection to this method of bookkeeping is not that the inventory is overvalued, because, in the absence of information in the problem, we must assume that the stock was billed to the branch at cost. The objection is that the unsold stock on hand at the branch is an asset of a somewhat different character from an account receivable. The accounts receivable represent a completed turnover and an asset which includes an earned profit. Merchandise inventory, on the other hand, represents merely an investment which has not been turned over and on which the profit, if any, is yet to be realized. An account receivable is more liquid than + +ILLUSTRATIVE PROBLEMS 109 + +merchandise inventory and thus it is advisable to state it separately in the accounts. + +The accounts with each branch should be separate and distinct from the accounts receivable and should be marked in such a way as to indicate that they represent merchandise stock on hand at the branch for sale. + +**Points Illustrated in Problem 37** + +(a) Deferred Credit. This problem illustrates the deferring of a credit in the case of the shipment of goods to a branch. There can be no income until the goods have been actually sold, although the cost of shipping and handling of the merchandise to the branch should be to a temporary account and not to a Sales account. (See Volume IV, Chapter IV, § 5.) + +(b) Inventory as an Asset. When merchandise is shipped from one branch to another, or from one warehouse to another, the value at which it is recorded on the books should not be increased, except possibly for the freight, carriage, and similar charges incurred in making the actual trans- fer. To bill a branch at a figure in excess of cost would be to anticipate a profit. (See Volume I, Chapter XI, § 18.) + +(c) Intercompany Profits. Where one branch transfers mer- chandise to another at an increase over cost, a situation analogous to intercompany profits arises. Such inter- branch, or interdepartment, or intercompany profits must be eliminated when the profit or loss of the business as a whole is sought. (See Volume IV, Chapter IV, § 6, and Chapter XXVIII, § 6.) + +--- + +**Problem 38** + +The Good Music Company sells pianos on the instal- ment plan. On January 2, 1914, Jones purchased a + +110 + +**ILLUSTRATIVE PROBLEMS** + +piano from the company for $375, to be paid as follows: +$25 down and the balance in quarterly instalments of +$50 each, bill of sale to be given on date of final pay- +ment. The piano cost the company $125. The first +instalments for 1914 were duly received, the last one +having been paid on December 31. + +Required: + +(a) Ledger accounts covering sale and payments +thereon. +(b) Yearly closing journal entry to credit year +with its proper proportion of the profit on +this transaction. + +(Wisconsin C. P. A. Examination.) + +**Solution to Problem 38** + + + + + + + + + + + + + + + + + + + + + + + + + + + + + + + + + + + + + + + + + + + + + + +
Jan. 2 Instalment Sales$375.00Jan. 2 Cash$35.00
Apr. 150.00
July 150.00
Oct. 150.00
Dec. 3150.00
Dec. 31 Balance150.00
$375.00$375.00
Jan. 1 Balance$150.00
+ +**INSTALMENT SALES** + + + + + + + + + + + + + + + + + + + + + + + + + + + + +
Jan. 2 Sales$35.00Jan. 2 Jones$375.00
Apr. 150.00
July 150.00 +
Oct. 150.00 +
Dec. 3150.00 +
Dec. 31 Balance150.00 +
$375.00$375.00
Jan. 1 Balance$150.00
+ +ILLUSTRATIVE PROBLEMS 111 + +**SALES** + + + + + + + + + + + + + + + + + + + + + + + + + + + +
Jan.Instalment Sales$ 25.00
Apr. 1"$ 50.00
July 1"$ 50.00
Oct. 1"$ 50.00
Dec. 31"$ 50.00
+ +$825.00 + +**CASH** + + + + + + + + + + + + + + + + + + + + + + + + + + + +
Jan.Jones$ 23.00
Apr. 1"$ 50.00
July 1"$ 50.00
Oct. 1"$ 50.00
Dec. 31"$ 50.00
+ +$825.00 + +Sales $825.00 +Piano Purchase $75.00 +Profit and Loss $150.00 +To credit Profit and Loss with $3/2 of $825. + +Points Illustrated in Problem 38 + +(a) Instalment Sales. The method of accounting for instalment sales is explained in Volume II, Chapter XVI, § 23. +(b) Deferred Credit. In this problem the profit on the instalment sale is recorded as earned only in the proportion that the cash collected bears to the total sale price. As the piano which cost $825 was sold for $375, a profit of $450 will result when the sale has been paid for. Of the $375, however, only $225 has been collected. Accordingly, $225/375 or $6/7 of the profit of $450 is recorded as earned during the period. The rest of the profit is deferred as explained in Volume IV, Chapter IV, § 4. + +112 +ILLUSTRATIVE PROBLEMS + +**Problem 39** + +On January 1, 1908, the condition of a small trading company was as follows: + + + + + + + + + + + + + + + + + + + + + + + + + + + + + + + + + + + + + + + + + + + + + + + + +
Assets:
Purchases and Fisteaux$2,000.00
Cash500.00
Notes Receivable3,000.00
Accounts Receivable5,000.00
Merchandise on Hand6,000.00
Capital Stock and Liabilities:
Capital Stock$5,000.00
Notes Payable3,000.00
Accounts Payable8,500.00
Savings500.00
_____________ $14,500.00
+ +During the month of January, the bookkeeper made all entries in the cash book and in the sales book, but made no journal entries and did not post his ledger. In addition to the entries appearing in the cash book and on the sales book, the following transactions took place during January: merchandise purchases on credit amounting to $6,000; notes payable amounting to $6,000 renewed; special allowance of $500 made to customers. + +The credit sales journal had two columns, one for the billed amounts and the other for the cost of the goods sold. The billed amount was $8,000, and the cost $5,000. + +The following statement gives a summary of the cash receipts and disbursements for January: + + + + + + + + + + + + + + + + + + + + + +
Cash Received:
Collected from Customers$4,000.00
Collected on Notes Receivable2,000.00
Collected on Merchandise Sold and not entered on sales book (cost $366)666.67
_____________ $6,666.67
+ +ILLUSTRATIVE PROBLEMS 113 + +Cash Payments: +Interest on Notes Payable $ 45.00 +Salaries 500.00 +Rent 900.00 +Sundry Expenses 300.00 +Accounts Payable 6,084.50 + +Prepare balance sheet, January 31, 1908, and statement of profit and loss, based on the book value of the merchandise. + +(Iowa C. P. A. Examination.) + +Solution to Problem 39 + +BALANCE SHEET +January 31, 1908 + + + + + + + + + + + + + + + + + + + + + + + + + + + + + + + + + + +
Assets
Furniture and Fixtures$8,000.00
Cash1,500.00
Notes Receivable6,000.00
Accounts Receivable8,500.00
Merchandise on Hand4,500.00
Total Assets$17,055.00
+ + + + + + + + + + + + + + + + + + + + + + + + + + + + + + + + + + + + + + +
Capital Stock and Liabilities
Capital Stock$8,000.00
Notes Payable6,000.00
Accounts Payable4,500.00
Sundry Expenses
Balance, January 1, 19088,500.00
Profit for January1,535.00
Total Liabilities.$17,055.00
+ +I + +114 +ILLUSTRATIVE PROBLEMS + +SCHEDULES SHOWING DETERMINATION OF SUNDRY BALANCE SHEET ITEMS + + + + + + + + + + + + + + + + + + + + + + + + + + + + + + + + + + + + + + + + + + + + + + + + + + + + + + + + + + + + + + + + + + + + + + + + + + + + + + + + + + + + + + + + +
Cash:
Balance, January 1$ 300.00
Received6,000.00$ 7,100.00
Paid6,045.00
Balance, January 31$1,053.00
Notes Receivable:
Balance, January 1$ 3,000.00
Collected$2,090.00
Balance, January 31$1,000.00
Accounts Receivable:
Balance, January 1$2,000.00
Net Charge Sales7,500.00$12,500.00
Received on Account4,000.00
Balance, January 31$8,500.00
Accounts Payable:
Balance, January 1$6,000.00
Purchases on Credit6,000.00$12,000.00
+
Payments:
By Cash$5,000.00
By Note(s)3,000.008,000.00

+ Balance, January 31 + $4,553.87 + + +The Notes Payable account shows a $8,353 balance on January 1, 1985, whereas the problem states that notes to the sum of $6,553 were renewed. It must be assumed that additional notes to the amount of $8,353 were given during January to trade creditors, and that, therefore, at January 31, 1985, the total amount of notes payable outstanding was $6,553, as stated in the above balance sheet. + +ILLUSTRATIVE PROBLEMS 115 + +PROFIT AND LOSS STATEMENT + +For the Month of January, 1908 + + + + + + + + + + + + + + + + + + + + + + + + + + + + + + + + + + + + + + + + + + + + + + + + + + + + + + + + + + + + + + + + + + +
Gross Sales$8,600.60
Less—Allowances300.00
Net Sales$8,100.60
Deduct—Cost of Goods Sold:
Inventory, January 1, 1908$4,000.00
Add—Purchases6,000.00
Less—Inventory, January 31, 1908$5,500.00
Gross Profit on Sales$2,600.60
Deduct—Expenses:
Salaries8 $500.00
Rent995.00
Sundry Expenses395.00
Profit from Operations$1,605.60
Deduct—Other Charges:
Interest on Notes Payable45.00
Net Profit$1,555.60
+ +The gross sales comprise $8,000 charge sales per sales book, and $600 cash sales per cash book. + +The inventory as of January 31 is found by deducting from the sum of goods on hand January 1 and purchases for the month the cost of goods sold as found in the sales and cash books. + +Points Illustrated in Problem 39 + +(a) Single-Entry Statements. The method of preparing statements from single-entry records and the points to be borne in mind in the determination of the financial condition and the profit or loss, are explained in Volume I, Chapter XXXV, §7. + +116 +ILLUSTRATIVE PROBLEMS + +(b) Interest on Notes Payable. This interest is stated on the profit and loss statement as a deduction from the profit due to the operations of the business. The theory underly- +ing this method of treatment is explained in Volume I, +Chapter IX, § 11. + +(c) Cost of Goods Sold. In this problem the cost of the goods sold is estimated daily by means of a price list or other record from which the information can be secured. Usu- +ally in a mercantile business the cost of goods sold is not exactly known until the end of a fiscal period, when it is calculated through the use of the closing inventory, as described in Volume I, Chapter IX, § 8. It is seldom that the keeping of a closing inventory is practicable unless required by a cost system. (See Volume III.) + +(d) Drawings by Proprietor. In this problem there were no drawings by the proprietor. This is an unusual situation because the usual practice is for the proprietor to with- +draw cash from the business on account of the profits earned by it. + +--- + +**Problem 40** + +The firm of Gray and Green has insured the life of Gray for $80,000, the policy being payable to the firm. +The annual premium is 8989.60. The policy does not have a cash surrender value until the end of the third year; at that time the surrender value is $82,150.02; fourth year $83,302.20; fifth year $83,567.43. The pre- +mium is paid at the beginning of the year. + +At the end of the fifth year Gray dies and the policy is paid to the firm in full. + +Show by journal entries how the transactions per- +taining to the above would be recorded. + +--- + +ILLUSTRATIVE PROBLEMS 117 + +**Solution to Problem 40** + +The insurance policy may be capitalized on some conservative basis, instead of treating each year's premium as an expense, because, unlike fire insurance, there is being created an asset which can be realized upon either by the death of the assured or by the surrender of the policy. + +The basis of capitalization is often the annual premiums paid to the insurance company, but a more conservative practice and one which is sound from an accounting standpoint is to capitalize the premiums at the cash surrender value of the policy, charging the difference between the premium and the increase in the cash surrender value to an expense account. + +In the above case the policy has no cash surrender value until the third premium has been paid, which brings up the question as to how to treat the first three premiums. Such premiums may be charged to expense and the policy capitalized at its cash surrender value at the end of the third year, crediting at the same time the partners' capital accounts. This method is objectionable for the reason that there is an abrupt change in the amount of insurance expense after the third year and likewise the capital accounts are suddenly increased due to a condition which has been in effect for three years. + +A better plan would seem to be to capitalize in each of the first three years, one-third of the cash surrender value of the policy at the end of that time, charging the difference between this value and the amount of the premium to expense; this method tends to equalize the amount of insurance expense during the time the policy + +A page from a book with text discussing insurance policies and their capitalization. + +118 + +**ILLUSTRATIVE PROBLEMS** + +is in force and also brings the policy on the books as an asset at the time it is taken out. + +By this method, the following entries would be made: + + + + + + + + + + + + + + + + + + + + + + +
First Payment:
Life Insurance Policy—Gray$ 716.87
Life Insurance Expense272.73
Cash$ 989.60
+ +**Second and Third Payments:** Same entry as for the first. + + + + + + + + + + + + + + + + + + + + + + +
Fourth Payment:
Life Insurance Policy—Gray711.58
Life Insurance Expense878.09
Cash$ 989.60
+ +**Fifth Payment:** Life Insurance Policy—Gray $705.05 / Life Insurance Expense $94.55 Cash $989.60 + +**End of Fifth Year:** Cash $50,000.00 Life Insurance Policy—Gray $50,000.00 Received cash in settlement from insurance company. + + + + + + + + + + + + + + +
Life Insurance Policy—Gray:$6,452.75
Gray, Capital$2,816.58
Green, Capital$3,816.57
+ +To credit each partner's capital account the increase in capital resulting from the excess of the insurance received over the capitalized value of the policy at the time of Gray's death. + +**Points Illustrated in Problem 40** + +(a) Deferred Debits. This problem illustrates the desirability of setting up deferred debits where expenditures of one + +ILLUSTRATIVE PROBLEMS 119 + +period involve expense applicable to a subsequent one. This is explained in Volume IV, Chapter II. For an explanation of "expenditures" and their classification into capital and revenue expenditures, see Volume I, Chapter X, § 4. + +(b) Adjustment of Capital. In this problem adjustment of the capital accounts is necessary in order to record the sudden acquisition of the life insurance cash. This sort of adjustment of capital where the increase does not result from current profits is explained in Volume IV, Chapter VII, § 4. + +(c) Distribution of Profits. As stated in Volume I, Chapter XXXII, § 6, profits and losses in a partnership are distributed equally in the absence of an agreement. In this case, as there was no agreement, the distribution is in equal parts. + +(d) Dissolution by Death. A partnership is dissolved upon the death of a partner unless there is some agreement binding upon his personal representatives. (See Volume I, Chapter XXXIII, § 3.) Notwithstanding the death of partner Gray, his share of the life insurance policy is credited to his account at the firm's office. The balance of that account, when finally adjusted, will measure his interest in the firm business. + +--- + +**Problem 41** + +A company with head office in Chicago and factory on South Bend, Indiana, conducts three selling branches in New York, San Francisco, and Montreal, which are supplied with goods from the factory, the invoices being sent out from the head office. + +The branches keep their own sales ledgers, send out monthly statements to customers, and receive cash + +120 +ILLUSTRATIVE PROBLEMS + +against their ledger accounts, which they remit weekly to Chicago. + +All branch expenses, including salaries and wages, are paid by the branches from petty cash accounts, kept at a fixed balance of $500, by draft on head office. + +The following information is supplied by the branches at December 31, 1913, summarizing the transactions of the previous six months: + + + + + + + + + + + + + + + + + + + + + + + + + + + + + + + + + + + + + + + + + + + + + + + + + + + + + + + + + + + + + + + + + + +
New YorkSan FranciscoMontreal
Rent and Taxes Paid8 $900.08 $175.008 $75.00
Sales for 6 mos. to Dec. 31, 191312,500.0011,800.0010,225.00
Salaries and Wages1,650.001,520.001,690.00
Returned Sales495.00100.00430.00
Allowance for Current Bad Debts59.0049.0039.00
Bad Debts125.0060.00
Cash Sales6,850.005,380.006,195.00
Cash Received from Customers Ledger Accounts.19,855.0019,826.009,135.00
Debtors, July 1, 1913 . . . . . . . . . . . . . . . . . . . . . . . . . . . . . . . . . . . . . . . . . . . . . . . . . . . . . . . . . . . . . . Debitors, Dec. 31, 19135,885.006,145.007,845.00
$2,725.00$7,435.00$7,975.00
Petty Cash, Dec. 31, 1913.
+ +Petty Cash, Dec. 31, 1913 +Stock, July 1, 1913 +Stock, Dec. 31, 1913 +Goods Received from Head Office +Faxed: + +Required: +(a) Branch accounts on head office books. +(b) Final general trial balance. +(c) Branch Profit and Loss accounts. + +(Illinois C.P.A. Examination.) + +ILLUSTRATIVE PROBLEMS 121 + +**Solution to Problem 41*** + +**(a)** + +**NEW YORK--CURRENT ACCOUNT** + + + + + + + + + + + + + + + + + + + + + + + + + + + + + + + + + + + + + + + + + + + + + + + + + + + + + + + + + + + + + +
Balance, July 1, 1913:Cash$8,500.00Cash, December 31, 1913$7,190.00
Debts.$8,500.00
Petty Cash300.00Debits.$7,190.00
Stock5,400.00 $9,770.00Petty Cash300.00
Best and Taxes.890.00Stock$2,650.00
Sales and Wages.1,265.0013,085.00
Goods from Factory.11,450.00
Net Profit.6,250.00
+
Total Debits.Total Credits.Total Difference.
$8,549.00$8,549.00$89,373.00
+ + SAN FRANCISCO--CURRENT ACCOUNTCash.$15,849.00Balance, December 31, 1913.$7,115.00Balance, July 1, 1913:Debits.$5,149.00Debits.$7,115.00Petty Cash.
DescriptionAmount
Cash.
DescriptionAmount
$8,549.00
DescriptionAmount
$8,549.00
DescriptionAmount
$89,373.00
<<<<<<<<<<<<<<<<<<<<<<
Description
Description
Description
Description
Description
Description
Description
Description
Description
Description
Description
Description
Description
Description
Description
Description
Description
Description
Description
Description
Description
Description
/tfoot/>tfoottfoot td { padding-left: 2em; padding-right: 2em; } tfoot td { padding-left: 2em; padding-right: 2em; } + +182 +ILLUSTRATIVE PROBLEMS + +(b) +A COMPANY +GENERAL TRIAL BAL +December + + + + + + + + + + + + + + + + + + + + + + + + + + + + + + + + + + + + + + + + + + + + + + + + + + + + + + + + + + + + + + + + + + + + + + + + + + + + + + + + + + + + + + + + + + + + + + + +
Head Office*New York
Dr.Ct.Dr.Ct.
1Investment in Branch House (July 1, 1913)$ 30,180.00
2New York--Current Account$ 83,120.00$ 17,100.00
3San Francisco--Current Ac- count$ 22,395.00$ 15,640.00
4Montreal--Current Account$ 85,415.00$ 15,250.00
5Factory$ 32,090.00
6Cash$ 47,990.00$ 6,720.00$17,100.00/$17,100.00
7T Petty Cash$ 8,330.00/$1,850.00
8Debtors$ 7,220.00
9Stock (July 1, 1913)$ 3,450.00
10Held for Sale$ 18,736.00
11Returned Sales$ 200.00
12Allowances to Customers50.00